Wrong Answer Box Flashcards

1
Q

A wealthy client ($10 million) wants her two adult children to learn to manage money. Rather than yearly gifts, she gifted each of them $1 million to invest in 2016. Your instructions were to invest the money as they order. Neither has asked you for advice. Due to market downturns, withdrawals, and poor timing of speculative stocks, one of the two children has run his account down to $150,000. He tells you to send him the balance of the account because he has found a for-sure investment. What should you do or what should you have done in the past?

A. You should have never taken him on as a customer

В. You should call Mrs. Cain

C. You should have requested that at account inception that duplicate statements be mailed to Mrs. Cain

D. You should have had the children provide an investment policy statement at inception

E. You should send the son the remaining account balance

A

E. You should send the son the remaining account balance

Mrs. Cain is not the client. You must do as the son asks. That eliminates B and C.

Answer D doesn’t answer the question. In answer A, Mrs. Cain asked you to take her children on as clients. By sending the remaining balance you have terminated the relationship. (See CFP Standards)

How well did you know this?
1
Not at all
2
3
4
5
Perfectly
2
Q

Bob Hayes consults with you, a CFP® practitioner, on investments. He opens a joint account with his wife. Bob reviews the account on a periodic basis with you by telephone. Unexpectedly, he instructs you to break the joint account into two separate accounts. Bob and his wife have decided to separate. He informs you which stocks go into his account and then tells you to place the remainder in her account. You realize he is keeping the better performing stocks. What should you do?

A. Follow Bob’s advice

B. Consult Bob’s wife before doing anything

C. Consult your compliance department

D. Terminate the relationship

E. Tell Bob you need to meet with him and his wife before you can respond

A

E. Tell Bob you need to meet with him and his wife before you can respond

Due to the JTWROS account, BOTH Bob and his wife are your clients. There is a conflict of interest in Answer A.

How well did you know this?
1
Not at all
2
3
4
5
Perfectly
3
Q

Index funds are appropriate if an investor believes in which of the following?

A. Strong form of the EMH

B. Weak form of the EMH

C. Semi-strong form of the EMH

D. Modern portfolio theory

A

A. Strong form of the EMH

EMH advocates a passive investment strategy (like a Modern portfolio theory is the selecting of an optimal combination of assets so that the investor secures the highest return for a given level of risk (active).

How well did you know this?
1
Not at all
2
3
4
5
Perfectly
4
Q

An exchange traded fund is most similar to a(n)?

A. Closed-End Fund

B. Open-End Fund

C. UIT

D. No-load Balanced Mutual Fund

E. Mostly an open-end fund but could be a closed-end fund

A

E. Mostly an open-end fund but could be a closed-end fund

An ETF may be an open-end or closed- end fund. Most ETFs are traded on a major exchange. It is possible to find a secondary market for UlTs units among brokers and dealers.

How well did you know this?
1
Not at all
2
3
4
5
Perfectly
5
Q

When compared against traditional index mutual funds, ETFs have which of the following advantages?

I. ETFs can be bought on margin.
II. ETFs can be sold short.
III. ETFs can be bought or sold throughout the trading day.
IV. Trading orders can include stop-loss and limit orders.

A

All of the Above

The ability to trade at market pricing, margin, and/or sell short are advantages to ETFs that mutual funds do not provide.

How well did you know this?
1
Not at all
2
3
4
5
Perfectly
6
Q

Modern portfolio theory expresses a risk-return relationship based on which of the following?

A. Weighted beta

B. Correlated coefficient

C. Alpha

D. Capital Market Line

E. Security Market Line

A

D. Capital Market Line

This is the best answer based on the following. The CML specifies the relationship between the risk and return on a variably weighted market portfolio consisting of all risky assets. When you read the material on chapter 7 page 1, this sounds like the CML. However, it is a difficult question to answer.

How well did you know this?
1
Not at all
2
3
4
5
Perfectly
7
Q

What is the most important consideration relative to the Markowitz efficient frontier?

A. Return

B. Covariance

C. Risk

D. Correlation

A

C. Risk

Answers A, B, and D play a supporting role in determining the efficient frontier. But answer C is the most fundamental, distinguishing characteristic of the efficient frontier.

How well did you know this?
1
Not at all
2
3
4
5
Perfectly
8
Q

The Random Walk Hypothesis suggests which of the following:

A. That technical analysts can outperform the market.

B. That fundamental analysts can outperform the market.

C. That the next price change of a stock is unrelated to the last price.

D. That security pricing reflects all known information.

A

C. That the next price change of a stock is unrelated to the last price.

If prices move randomly, technical analysis is useless.

Weak form is related to, but not identical with, random walk; therefore, fundamental analysis could be an answer.

Answer C is the definition of random walk, and Answer D is the definition of EMH.

How well did you know this?
1
Not at all
2
3
4
5
Perfectly
9
Q

Which of the following statements is true about Dow Theory?

A. The theory considers EMH principles.

B. It presumes that the most important price movement is inter-day fluctuations.

C. It is a method that is presumed to identify the top of a bull market and the bottom of a bear market.

D. It presumes that primary price movements are based on Modern Portfolio Theory.

A

C. It is a method that is presumed to identify the top of a bull market and the bottom of a bear market.

The Dow Theory contradicts Modern Portfolio Theory and the EMH. It is based on trends, not day to day fluctuations.

How well did you know this?
1
Not at all
2
3
4
5
Perfectly
10
Q

In what way is a financial ratio considered to be the most useful?

A. A single ratio across different industries over time

B. A single ratio within the same industry over time

C. Several ratios across different industries over time

D. Several ratios with the same industry over time

A

D. Several ratios with the same industry over time

To identify the most promising stock within a particular industry, several ratios within the same industry can be compared.

One ratio by itself means little (Answer B), but several ratios together may give a clear picture of the firm’s strengths and weaknesses.

Then they may be compared at a given time for several firms within the same industry.

How well did you know this?
1
Not at all
2
3
4
5
Perfectly
11
Q

Which of the following is true about the arbitrage pricing theory?

A. Security movements are explained by a relationship between risk and return.

B. The expected value of each factor is zero.

C. Pricing of securities in different markets can differ for significant lengths of time.

D. Unexpected changes in inflation and anticipated shifts in risk premium will influence security prices.

A

B. The expected value of each factor is zero.

APT argues that unanticipated shifts in risk premium will influence security prices. Answer D says anticipated shifts. This is incorrect for APT.

When a factor is zero, the factor has no impact on the return because it is expected or anticipated.

How well did you know this?
1
Not at all
2
3
4
5
Perfectly
12
Q

Binomial option pricing is a __________________ model

A. pricing
B. valuation
C. volatility
D. variability

A

B. valuation

Like the Black-Scholes model, binominal options pricing is a valuation model.

Prices are established through the action of buyers and sellers; investors and analysts use valuation models to estimate what those prices should be.

How well did you know this?
1
Not at all
2
3
4
5
Perfectly
13
Q

When John dies, he wants his wife to have annual income of $36,000 that will increase with inflation (4%). She can realize an after-tax return of 7%. Using a capital retention calculation, how much insurance he should purchase to cover her needs?

A. $514,285

B. $852,000

C. $1,200,000

D. $1,236, 000

E. $1, 248,007

A

Answer: D

This is not a time value question because there is no third variable - time.

  1. 7% - 4% = 3%
  2. $36,000 / .03 - $1,200,000
  3. Plus beginning of year 1 money +$36,000

Total $ 1,236,000

How well did you know this?
1
Not at all
2
3
4
5
Perfectly
14
Q

New clients walk into your office. They have a baby. They have done no planning.
They own limited assets, carry a reasonable amount of credit card debt, and a fair amount of college debt. They are confused and seeking your help. What should they do first?

A. Buy medical insurance.

B. Buy a lot of term life insurance.

C. Establish a 6 month emergency fund.

D. Consult an attorney to establish a guardianship because they are not married.

E. Establish a Coverdell ESA for grade school.

A

A. Buy medical insurance.

Answer A addresses the most serious and immediate concern. Nothing says that they are married.

How well did you know this?
1
Not at all
2
3
4
5
Perfectly
15
Q

What are the obligations of an agent to the principal?

I. Loyalty
II. Accuracy
III. Full disclosure
IV. Honesty

A. All of the above
B. ІІ, III
C. III, IV
D. IV

A

A. All of the above

All the answers are correct, even loyalty. For example, a company (the principal) may forbid an agent to write a certain type of coverage. However, if the agent wrote that kind of coverage for a friend, the company would be liable for any loss that occurred.

How well did you know this?
1
Not at all
2
3
4
5
Perfectly
16
Q

Ted and Hallie Fisher have been your clients for 25 years. They recently told you that due to medical problems they both have to move into an assisted living facility. They have about $500,000+ in their investment account. Most of that came from to the sale of their home two years ago. Their Social Security retirement payments total about $1,500 a month due to taking benefits at age 62. What should you recommend they do?

A. Determine how much the assisted living facility will cost.

B. See an elder care attorney.

C. Execute a living will.

D. Name a family member who can handle their financial affairs.

E. Reposition their account for more liquidity and income.

A

B. See an elder care attorney.

The Fishers need expert legal advice before making decisions about assisted living and other matters.

How well did you know this?
1
Not at all
2
3
4
5
Perfectly
17
Q

A company owns whole life policies on key employees. The company is considering borrowing the cash value of the policies for business reasons. The company must to pay the interest on the loans. Is the interest an eligible business deduction?

A. No, after 1986, life insurance interest isn’t deductible.

B. No, it is not a normal business expense.

C. Yes, but it is limited to a $50,000 loan on each policy.

A

C. Yes, but it is limited to a $50,000 loan on each policy.

A business may deduct a limited amount of interest paid on contracts covering a “key person” to the extent that each loan doesn’t exceed $50,000.

How well did you know this?
1
Not at all
2
3
4
5
Perfectly
18
Q

Phillip was V. P. of sales with Glamour, Inc. Glamour owned an endorsement method split-dollar policy on his life. He decided to buy the policy from Glamour and transfer it to a new company that he was starting with two other people (33% each). If Phillip dies suddenly, will the death benefit be included in his estate?

A. No, this is what “transfer for value” avoids.

В. No, Dan gifted the policy to the new corporation (not a sale).

C. Yes, if he dies within three years of the transfer.

D. No, it will only be included if he changed the beneficiary.

A

C. Yes, if he dies within three years of the transfer.

At the time of the transfer, Phillip had an incident of ownership (right to name the beneficiary). The three-year inclusion rule is in effect. Answer A is true, but Phillip buying his policy doesn’t trigger a “transfer for value”.

How well did you know this?
1
Not at all
2
3
4
5
Perfectly
19
Q

Which type of employer group coverage is not tax deductible by the employer?

A. Group life

B. Group legal

C. Code section 125

D. Group disability

A

C. Code section 125

Flexible spending accounts (Section 125) are funded solely by the employee. The other plans are normally paid for by the employer (tax deductible). Code Section 125 is not group coverage, it is a cafeteria plan.

How well did you know this?
1
Not at all
2
3
4
5
Perfectly
20
Q

What type of company is most likely to use stop-loss coverage to partially self-insure its employee medical insurance program?

A. Large companies

B. Medium-to-large companies

C. Companies with as few as 100 employees

D. Not-for-profit associations

A

C. Companies with as few as 100 employees

For example, a relatively small company could self-insure employee claims to $250,000 per claim.

Claims above $250,000 in aggregate would then be paid by an insurance company.

Claims under $250,000 in aggregate would be financed by the employee, employer contributions, and potentially reduced health insurance premium costs.

How well did you know this?
1
Not at all
2
3
4
5
Perfectly
21
Q

Which of the following is not a source of information generally used during the underwriting process?

A. Information from the broker

B. Cost-benefit analysis

C. Physical examinations

D. Investigations

A

B. Cost-benefit analysis

The applicant, rather than the underwriter, would perform cost-benefit analysis. The other answers are correct.

How well did you know this?
1
Not at all
2
3
4
5
Perfectly
22
Q

Bill Yates was involved in a not-at-fault auto accident. He was hurt, and his car was damaged. A hit-and-run vehicle left the scene of the accident before Bill could get the driver’s name or license number.

Under which parts of his auto policy (he has A, B, C, and D coverage) can he collect?

I. Bodily injury/property damage
II. Medical payments
III. Uninsured motorist
IV. Collision
V. Other than collision

A. I, II, III
B. II, III, IV
C. II, V
D. III, IV
E. IV

A

B. II, III, IV

Medical payments and collision coverages apply. Answer III protects him when he gets sued, but Bill was not at fault. Bill must file a claim against his own insurer to collect under the uninsured motorist provision. Basically, he sues himself. This presumes the hit-and-run driver cannot be identified. Item I is incorrect because Coverage A would apply if Bill were at fault and if the other party were injured or his/her property were damaged.

How well did you know this?
1
Not at all
2
3
4
5
Perfectly
23
Q

Which of the following statements about umbrella liability insurance policy coverage are true?

I. It provides excess coverage when the limits of the insured’s basic liability coverage are adequate.

II. It provides excess coverage when the limits of the insured’s basic liability coverage are inadequate.

III. It provides broader coverage than basic underlying policies.

A. I, II, III
B. I, III
C. II, III
D. I, II

A

A. I, II, III

The material shows that the umbrella even provides benefits when the client has inadequate coverage. The insured is responsible for gaps in coverage. For the CFP Board Certification Examination, the client should have umbrella liability coverage.

How well did you know this?
1
Not at all
2
3
4
5
Perfectly
24
Q

Which of the following employer paid insurance premiums or costs are tax-deductible?

I. Business owner’s insurance
II. Key employee life insurance
III. Workers’ compensation insurance
IV. Federal unemployment tax
V. Directors and officer’s liability insurance

A. I, III, IV, V
B. I, II, III
C. II, IV
D. III, IV, V
E. III, V

A

A. I, III, IV, V

Premiums for I, III, IV, and V are business deductions. Unemployment benefits are taxable income. Answer Il is not tax-deductible. Federal unemployment tax is FUTA and is deductible.

How well did you know this?
1
Not at all
2
3
4
5
Perfectly
25
Q

For how many days does the COBRA election period last?

A. 30 days after termination

B. 30 days after the actual notice of the event to the qualified beneficiary by the plan administrator

C. 60 days after termination

D. 60 days after the actual notice of the event to the qualified beneficiary by the plan administrator

A

D. 60 days after the actual notice of the event to the qualified beneficiary by the plan administrator

The COBRA election period starts with the date of the notification and lasts for 60 days.

How well did you know this?
1
Not at all
2
3
4
5
Perfectly
26
Q

When Phyllis became ill (but not disabled) for a few weeks, she and her husband experienced a negative cash flow.

Although both are covered under disability insurance plans, they feel that the amount of existing coverage will be insufficient to maintain their current lifestyle. What would a CFP® professional most likely advise?

A. Both spouses should increase their existing individual disability plans to maximum benefits permitted by law.

B. Both spouses should take out additional life insurance and add disability waiver of premium
endorsements.

C. Both spouses should take out hospital and surgical indemnity policies.

D. Both spouses should take out an LTC policy with home health care coverage.

E. Both spouses should apply for additional individual disability policies with SIS benefits and guaranteed insurability options.

A

E. Both spouses should apply for additional individual disability policies with SIS benefits and guaranteed insurability options.

Both spouses need additional policies. One cannot increase the benefits on an existing policy. Even with a guaranteed insurability rider, a new additional policy would need to be issued.

How well did you know this?
1
Not at all
2
3
4
5
Perfectly
27
Q

Otis would like to purchase a disability insurance policy, but premium cost is a problem. What can he do to reduce the premium costs?

I. Increase the elimination period
II. Delete the presumptive disability coverage
III. Delete the continuance provision
IV. Not buy the COLA rider
V. Purchase a guaranteed renewable policy

A. I, IV, V
B. II, III
C. I, III, IV
D. II, III, V
E. IV, V

A

A. I, IV, V

Otis can reduce the premium by electing a longer elimination period and/or choosing a policy with a guaranteed renewable rather than non-cancellable provision.

The presumptive disability clause and the continuance provision clause are included in the policy.

COLA is a rider (extra cost).

How well did you know this?
1
Not at all
2
3
4
5
Perfectly
28
Q

Lester owns a whole life contract. Unknowingly, he ‘MEC’ed the contract by paying too much premium. Lester is now concerned about the dividends being paid by the contract. Under what types of distributions are dividends paid by this contract taxable?

I. They are received in cash.
Il. They are used to reduce premium.
III. They are used to purchase paid-up additions.
IV. They are kept with company and accumulate with interest.
V. They are used to pay off a policy loan.

A. Dividends from participating policies are never taxable because they represent return of basis.

B. All of the above

C. I, II, V

D. III, V

E. II, V

A

C. I, II, V

Participating policy dividends from MECs become subject to current-year income tax when they are received in cash or used like cash to reduce upcoming premiums or to pay back policy loans.

The policy dividends aren’t taxable when they accumulate with interest. However, the interest paid on the dividends is taxable.

The contract is a MEC.

How well did you know this?
1
Not at all
2
3
4
5
Perfectly
29
Q

A non-key employee in discriminatory group term life insurance can exclude the taxable cost of the first of coverage?

A. $0

B. $50,000

C. One-time salary

D. Two-times salary

A

B. $50,000

The non-key employees may exclude the premium that is attributable to the first $50,000 of coverage in a group-term life insurance program.

How well did you know this?
1
Not at all
2
3
4
5
Perfectly
30
Q

Which of the following group type insurance policies do not offer a conversion feature?

I. Life
Il. Health
III. Short-term disability
IV. Long-term disability

A. I, II
B. II, III
C. II, IV
D. III, IV
E. IV

A

D. III, IV

Life and health group insurance coverage provides conversion features. See end of Chapter 4 Insurance for group health convertibility.

Group disability is not convertible.

How well did you know this?
1
Not at all
2
3
4
5
Perfectly
31
Q

Jack is age 50 and divorced. He no longer has to pay alimony or child support. His son is 28 and on his own. Jack has two life insurance policies: $60,000 of group term (son is the beneficiary) and $250,000 of whole life (estate is the beneficiary). He has been paying $3,000 of premium annually for 10 years on the $250,000 policy and can afford to continue to pay premiums. He wants to maximize his retirement benefits, and he believes he does not need as much life insurance as he now carries.

When Jack retires, should he convert his group term policy? If he converts, which kind of policy can he convert to? The Question 14 facts remain unchanged except Jack is now age 65.

A. Convert to decreasing term

B. Convert to a whole life policy

C. Terminate the coverage at age 65

D. Convert to a variable annuity

A

B. Convert to a whole life policy

Jack does not need much life insurance at age 50. Answer 14 does not indicate no need. The $60,000 of group term is not a large policy. Conversion to whole life seems to fit the question best. He cannot convert a group term policy to an annuity.

How well did you know this?
1
Not at all
2
3
4
5
Perfectly
32
Q

Which of the following statements concerning Medicare Part A benefits is correct?

A. To be eligible, the insured must be at least age 65.

B. Home health care visits are covered under Part D.

C. Hospice care is specifically excluded.

D. Benefits are subject to both a deductible and co-payments provisions for inpatient hospital care.

A

D. Benefits are subject to both a deductible and co-payments provisions for inpatient hospital care.

Part A benefits are subject to a deductible and copayments. Disabled workers, for example, can be covered under age 65. Hospice care is covered under Part A. Medicare Part D is for drugs.

How well did you know this?
1
Not at all
2
3
4
5
Perfectly
33
Q

Sam’s knee problem makes him nervous about his disability insurance. Sam is a regional sales manager for Hyper-Tech, Inc. He earns $200,000 in salary plus a $200,000 (estimated) bonus. This is a perfect job for him, and he doubts he can duplicate this job and money with any other employer. How should he cover himself for disability considering his current coverage?

I. He should opt-out of his current group disability plan and purchase the maximum amount of individual disability coverage.

II. He should supplement his group disability plan with a new individual plan with own occupation coverage up to age 65.

III. He should build up an emergency fund and self-insure for a reasonably long waiting period with a new individual policy.

IV. He should buy a guaranteed insurability rider in the new individual policy. He can exercise the option if he loses his job for any reason.

V. He should buy a partial disability rider in the new individual policy. This benefit will be paid until age 65
if he is partially disabled.

A. I

В. II, III, IV, V

C. II, III, IV

D. II, III

E. lI

A

D. II, III

If possible, Sam should supplement his current plan. The group plan definition of disability is his own occupation for 5 years. He doesn’t have much spendable income. It makes sense to keep the group plan and build up his emergency fund.

Answer IV is false. To exercise a guaranteed insurable option, he must show proof of higher earnings.

Answer V is false. This defines the residual disability rider. The partial disability rider is normally free.

How well did you know this?
1
Not at all
2
3
4
5
Perfectly
34
Q

Mr. and Mrs. Apple, New Jersey residents, have a daughter who attends Cornell University in New York State.

They give her a car while she is a college student. After college, she takes a job and will live in Manhattan. Is her car still eligible for coverage under the parents’ PAP policy?

A. No

B. Yes

C. It depends on whether she can be claimed as a dependent.

D. She will not be protected for out of state coverage.

A

A. No

While away at college, the daughter is still considered a resident of her parents’ home (a dependent). After taking a job and residing in New York, she must license and insure the car in New York.

How well did you know this?
1
Not at all
2
3
4
5
Perfectly
35
Q

Sharon purchases a home for $330,000. The value of the land is $50,000. The mortgage is $260,000. The property is covered for fire-related perils by XYZ Insurance Company to $225,000 with an 80% coinsurance provision and a $1,000 deductible. Does Sharon have sufficient coinsurance coverage for the home?

A. Yes

B. No

C. The client may suffer a penalty due to a partial loss.

D. If the property is a total loss, the mortgage will not be completely paid off.

A

Question 35: A

The replacement value of the home is $280,000. The insurance is $225,000. Sharon is insured for 80%. 80% of $280,000 is $224,000. The mortgage does not factor into this question.

How well did you know this?
1
Not at all
2
3
4
5
Perfectly
36
Q

Sally (age 35) and Hy (age 36) want to fund a buy-sell agreement with life insurance. The insurance carrier is willing to offer life insurance on Hy but only with a rating (extra premium). Which option should Sally and Hy choose to achieve the greatest mutual benefit?

A. They should use a cross-purchase type buy-sell: They should each purchase a level-term policy even if
Hy is rated. Hy may have to pay more.

B. They should use a cross-purchase type buy-sell. They should each purchase variable universal life on each other’s lives. Some of Hy’s premium rating (out-of-pocket cost) could be reduced by wise investment choices.

C. They should use a redemption agreement type buy-sell. Their company should purchase 2 universal life policies.

D. They should use a redemption agreement type buy-sell. Their company should purchase 2 limited-pay whole life policies. The policies would be creditor proof.

A

B. They should use a cross-purchase type buy-sell. They should each purchase variable universal life on each other’s lives. Some of Hy’s premium rating (out-of-pocket cost) could be reduced by wise investment choices.

In a cross-purchase arrangement, Sally would purchase Hy’s policy (the higher premium.) She would pay the higher premium. With good investment performance the variable policy may be less costly. Answer C is wrong.

Corporate ownership of policies makes them subject to the corporation’s creditors. A cross-purchase agreement gives the survivor a tax advantage (step-up in basis). Hy has a shortened life expectancy, step-up in basis is more important to him than minimizing premiums.

How well did you know this?
1
Not at all
2
3
4
5
Perfectly
37
Q

Which of the following statements is/are true about John’s group term life insurance contract?

I. He can make an absolute assignment to Helen as the beneficiary and remove the policy from his estate.
II. He owns the contract.
III. He makes no contribution to the premium.
IV. The $120,000 is included in his estate although it passes through to Helen under the marital deduction.
V. At John’s death, Helen must pay income tax on the $70,000 excess death benefit.

A. I, II, III, IV
B. I, II, IV
C. I, III, IV
D. IV, V
E. IV

A

Question 46: C

Answer l is true because making an absolute assignment of a policy will remove the policy from his estate subject to the 3-year rule.

Answer Il is false. The company owns the policy.

Answer Ill is true. He is charged with income (W2) for the excess benefits (above $50,000), but he doesn’t pay the premium.

Answer V is false.
John will be charged with the Table I cost on premiums attributable to benefits above $50,000 ($120,000 -
50,000 = $70,000). Because he is charged with the premium, the benefits received are income tax free.

NOTE: Although the company owns the policy, because he named the beneficiary (Helen), according to the Internal Revenue Code he holds “incidents of ownership” in the policy for federal estate tax purposes.

How well did you know this?
1
Not at all
2
3
4
5
Perfectly
38
Q

What are Bob’s options for health insurance?

I. He can get up to 36 months of COBRA.
II. He can convert the group coverage to an individual health plan.
III. He can elect a HIPAA option.
V. He might be able to continue under his father’s plan.
V. He can apply for group insurance when he starts his own business.

A. I, III, IV
B. II, III, IV, V
C. III, IV, V
D. III, IV
E. V

A

B. II, III, IV, V

COBRA is not an option because the business has only 10 employees. He has a conversion option under his father’s policy. (No evidence of insurability is required.) HIPAA is an option. As long as he is a dependent of his father, Bob can continue under his father’s plan until age 26. If Bob starts his own business, he can apply for group insurance. Under Affordable Care Act, Bob cannot be denied coverage due to a pre-existing condition.

How well did you know this?
1
Not at all
2
3
4
5
Perfectly
39
Q

In case Mark or Mary dies, what type of advanced business planning should they implement?

I. Entity purchase buy-sell
II. Cross-purchase buy-sell
III. Key person life insurance on Mark
IV. Split-dollar life insurance on Mark
V. Non-qualified deferred comp for Mark

A. I, III, IV, V

B. II, III, IV, V

C. I, III

D. II, III

E. III, IV, V

A

Question 59: D

The company is owned 50/50. Mark’s children do not like Mary. Mark and Mary should choose a cross-purchase buy sell agreement. The business should buy a key-employee policy on Mark. The agency relies on Mark’s selling ability. Split dollar and non-qualified deferred compensation reflect personal planning rather than business planning. Mark already has an existing split-dollar policy. Cross-purchase is a better choice than entity purchase for Mary. She can buy Mark’s stock at his death. Otherwise, his children will get his stock.

How well did you know this?
1
Not at all
2
3
4
5
Perfectly
40
Q

Mark is concerned about becoming disabled and his company subsequently failing. If he purchases a BOE policy (business overhead expense), which of the following statements is true?

A. The premium is not deductible, but the benefits are received tax- free.

B. The premium is deductible, but the benefits are taxable income to the business.

C. The premium is deductible, and the benefits are received tax-free.

D. The premium is not deductible, and the benefits are taxable income.

E. Only self-employed persons can purchase BOE policies.

A

A. The premium is not deductible, but the benefits are received tax- free.

The corporation cannot deduct the premium, but it can exclude the benefits from its gross income.

How well did you know this?
1
Not at all
2
3
4
5
Perfectly
41
Q

What type of benefit plans can Richard install at RC, Inc. that

  1. Will not cost RC, Inc. very much money and
  2. Will help retain employees?

I. Nonqualified deferred compensation
II. Section 125 plan
III. Endorsement method split-dollar life insurance
IV. Group life insurance up to $50,000 of death benefits
V. Group dental plan (basic benefits)

A. I, II, IV, V
B. II, III, IV, V
C. I, III
D. II, IV, V
E. II

A

D. II, IV, V

Fringe benefits like a Section 125 plan, group life insurance up to $50,000 death benefit, and group dental are relatively inexpensive. NQDC is too expensive to implement.

How well did you know this?
1
Not at all
2
3
4
5
Perfectly
42
Q

Dr. Jeffrey is retired but bored. He is considering purchasing an antique refurbishing business. The business experienced marginal profits in the past. He has a knowledge of antiques and plans to upgrade the equipment and building. He feels that after losing money (due to upgrades), the business will be profitable. Dr. Jeffrey plans to put up all the capital.

Which of the following business forms is the most appropriate?

A. Sole proprietorship
B. Limited partnership
C. Personal Service Corp.
D. S corporation
E. C corporation

A

D. S corporation

Dr. Jeffrey needs the limited liability associated with the corporate form. With an S corporation, he can take losses up to his investment (basis).

How well did you know this?
1
Not at all
2
3
4
5
Perfectly
43
Q

When he files his Form 1040, Thomas will report the following income and losses for the year:

  • $5,000 loss from ownership (%) in a limited partnership
  • $10,000 loss from active participation in a strip shopping center (100% ownership)
  • $8,000 loss from a 10% interest in an S-corporation in which he is a manager
  • $75,000 of salary as the manager of the S-corporation

What is Thomas’ adjusted gross income for the current tax year?

A. $52,000
B. $57,000
C. $65,000
D. $70,000
E. $75,000

A

B. $57,000

Salary ($75,000) - [Active participation (-10,000) + S-Corp loss (-8,000)] = AGI ($57,000).

The exception to the
passive rules is active participation in real estate.

The S-Corp income is active. He manages the business.

How well did you know this?
1
Not at all
2
3
4
5
Perfectly
44
Q

Question 2

Joe Silver has active income of $300,000 per year and substantial unused passive losses from a non-publicly traded limited partnership.

He would like to find an investment that would allow him to utilize his passive losses. Which of the following are the most appropriate investments for Joe?

I. An active participation rental real estate activity generating income on the Schedule E

II. A master limited partnership (MLP) generating income

Ill. Certificates of deposit generating portfolio income

IV. A non-publicly traded limited partnership generating income

A. I,Il
B. I, IV
C. III, IV
D. I, II, IV
E. I, II, III, IV

A

B. I, IV

Schedule E income from active participation in rental activities can be used to offset other passive losses.

The non-publicly traded limited partnership losses may not offset income from a publicly traded partnership (the MLP) or portfolio income.

All of the choices include Answer I. Knowing IV was right leads to the correct response to the question.

MLPs are publicly traded

How well did you know this?
1
Not at all
2
3
4
5
Perfectly
45
Q

Which of the following phrases best applies to the AMT exemption?

A. Subject to phaseout
B. Usable by corporations only
C. Deductible
D. Always deductible

A

A. Subject to phaseout

The AMT exemption is subject to phaseout rules.

How well did you know this?
1
Not at all
2
3
4
5
Perfectly
46
Q

Which of the following strategies should be considered to avoid or reduce exposure to the AMT tax?

A. Exercise of ISOs in one year
B. If you are the owner of a small corporation, pay yourself a large bonus
C. Defer paying property taxes until next year
D. Buy private activity municipal bonds rather than public purpose municipal bonds

A

B. If you are the owner of a small corporation, pay yourself a large bonus

Increasing 1040 income tax decreases AMT exposure.

Answer D should read: Buy public purpose municipal bonds (because the interest is not a preference item rather than private purpose bonds (because the interest is a preference item).

How well did you know this?
1
Not at all
2
3
4
5
Perfectly
47
Q

Which of the following are preference items or add-back items for purposes of the individual alternative minimum tax?

I. Qualified private-activity municipal bond interest
II. The property tax itemized deduction
Ill. The excess of percentage depletion over the property’s adjusted basis
IV. Cost depletion deductions

A. I, III
B. I, IV
C. I, II, III
D. II, III, IV
E. All of the above

A

C. I, II, III

Cost depletion is not a preference item. Property tax is not a deduction for the AMT. It is added back to calculate the AMT (an adjustment).

How well did you know this?
1
Not at all
2
3
4
5
Perfectly
48
Q

A partnership trades an old piece of equipment having an adjusted basis of $10,000 plus cash of $2,000 for a new machine with a fair market value of $15,000. What is the recognized gain, and what is the basis of the new machine?

A. Gain $0; Basis: $12,000
B. Gain $0; Basis: $10,000
C. Gain $2,000; Basis: $15,000
D. Gain $3,000; Basis: $12,000

A

A. Gain $0; Basis: $12,000

This is NOT a 1031 exchange. The old basis and the cash become the new basis. 1031 exchanges are no longer available for equipment but only for real property.

How well did you know this?
1
Not at all
2
3
4
5
Perfectly
49
Q

Tim and Maggie Butler, who are married, sold their residence in June.

The realized gain over the eight years they owned the home is $350,000. Instead of buying a new home, they decide to rent a condo. Which of the following statements is true?

A. The gain must be reported on the tax return for the year of the sale.
B. There is no taxable gain; therefore, no tax forms need to be filed.
C. The amount of the gain is more than exclusion. No tax forms need to be filed.
D. A Schedule D and Form 2119 must be filed.

A

B. There is no taxable gain; therefore, no tax forms need to be filed.

The $350,000 realized gain is completely excluded by the $500,000 exclusion. No return needs to be filed because there is no recognized gain.

How well did you know this?
1
Not at all
2
3
4
5
Perfectly
50
Q

Nick owns a warehouse with a fair market value of $200,000 and an adjusted basis of $50,000. He wants to acquire Jim’s strip shopping center which has a fair market value of $300,000 and adjusted basis of $100,000. In the exchange, Nick will pay Jim $75,000.

What is the amount of gain realized by Nick in the exchange?

A. $75,000
B. $125,000
C. $175,000
D. $300,000

A

C. $175,000

Total value received minus new adjusted basis

FMV of property acquired
($300,000) + Boot ($0) = $300,000

[Minus]

Adjusted basis of property ($50,000 $75,000) = $125,000

How well did you know this?
1
Not at all
2
3
4
5
Perfectly
51
Q

Lance creates an irrevocable life insurance trust that will pay income to his ex-wife for life and then to his children.

Lance transfers a $1,000,000 term policy and $100,000 of high yield bonds to the trusten te income from the bonds will be used to pay the premiums on the policy, and all remaining income will be paid to family members.

Regarding the income from the trust, which of the following statements is correct?

A. During Lance’s lifetime, the income of the trust will be taxable to Lance.

B. During Lance’s lifetime, income from the trust will be taxable at the trust rates.

C. During Lance’s lifetime, the income of the trust will be split with the amount paid for insurance premiums taxable to the trust and the amount paid to the family members taxable to Lance.

D. The income from the trust will be taxable to the beneficiaries under the beneficial enjoyment rule.

A

A. During Lance’s lifetime, the income of the trust will be taxable to Lance.

This grantor trust is tainted because trust income will be used to pay life insurance premiums. Per the question only a portion of the trust income is being used to pay the premiums on the policy. The remainder is being paid to family members and will be taxable to them (DNI principle).

How well did you know this?
1
Not at all
2
3
4
5
Perfectly
52
Q

Under a divorce settlement, Mr. O’Toole was required to establish a trust to provide support for his minor children. The income generated by the trust will be taxed to which of the following parties?

A. Mr. O’Toole
B. The trust
C. Mr. O’Toole’s children
D. Mrs. O’Toole as child support payments

A

A. Mr. O’Toole

If the trust income is used to satisfy a grantor’s legal support obligation, the trust income will be taxed to the grantor.

How well did you know this?
1
Not at all
2
3
4
5
Perfectly
53
Q

Pete operates his landscaping business as a sole proprietorship. The business produces annual net earnings of $400,000. Pete read an article in the Sunday paper about incorporating to limit a business owner’s personal liability. He comes to you for advice. Which of the following statements would be proper advice for a CFP professional to tell Pete?

I. Pete should not incorporate his business because the top corporate income tax bracket is 21%.

II. A limited partnership would also protect him from liability.

III. An S corporation would save accounting costs because it does not have to file income tax returns.

IV. Pete can reduce his current income tax liability by splitting his income between himself and a C corporation.

V. If he gives them stock, an S corporation could allow Pete to shift income to his children.

A. II, III
В. I
C. II, V
D. IV, V
E. III, IV

A

D. IV, V

The corporate tax rate is 21%. As an active participant (general partner) the limited partnership would not limit Pete’s personal liability relative to the business. If Pete took a salary of $400,000, his marginal tax bracket is 35%. This way Pete could reduce his income tax liability. An S corporation could shift income.

How well did you know this?
1
Not at all
2
3
4
5
Perfectly
54
Q

Dr. K, Dr. L, and Dr. M own a successful medical corporation. At year-end, a profit of $150,000 is left in the corporation. How will the earnings be taxed?

A. To the individual doctors at their tax rate.
B. At the corporate graduated rates.
C. $100,000 at 20%.
D. At a flat 21%.

A

D. At a flat 21%.

HALE (Health) corporations are PSCs and taxed at 21%.

A PSC with accumulated earnings exceeding $150,000 will be exposed to a 20% accumulated earnings tax.

How well did you know this?
1
Not at all
2
3
4
5
Perfectly
55
Q

Which of the following filing statuses could help a business owner reduce his/her taxes?

A. File as a self-employed individual
B. File as an S corporation (Form 1120S).
C. File as a regular corporation (Form 1120).
D. File as an LLC.

A

Question 8: C

Answers A, B and D are conduit entities.

A regular corporation will provide the owner with a separate tax entity.

Money left in a corporation is taxed at 21%.

How well did you know this?
1
Not at all
2
3
4
5
Perfectly
56
Q

The OKA Corporation owns 25% of the stock of DEC Corporation. For the current tax year, OKA receives $10,000 in dividends from DEC. What is the amount of OKA’s dividend-received deduction?

A. $0
B. $2,000
C. $2,500
D. $6,500
E. $10,000

A

D. $6,500

65% of the dividend received is excluded because OKA owns 20% or more of DEC. Otherwise, the deduction is limited to 50% of the dividend received when a company owns less than 20% of the paying company.

How well did you know this?
1
Not at all
2
3
4
5
Perfectly
57
Q

If a company elects the FIFO method of inventory control, how will the inventory be reflected on the balance sheet?

A. It will be understated.
B. It will reflect current cost.
C. It will be based on the accrual method.
D. It will be based on the cash method.

A

B. It will reflect current cost.

The FIFO method of inventory valuation reflects current cost.

How well did you know this?
1
Not at all
2
3
4
5
Perfectly
58
Q

Which type of business entity cannot adjust future years’ income due to prior year(s) NOL?

A. Regular corporations
B. S corporations
C. Estates
D. Trusts

A

S corporations cannot utilize NOLs because they already pass-through annual losses.

How well did you know this?
1
Not at all
2
3
4
5
Perfectly
59
Q

Which of the following is (are) true about a personal casualty loss (federally declared disaster)?

I. If the property subject to the loss is not fully insured, the casualty loss is not deductible.
Il. The casualty loss is deducted on the Schedule A.
Ill. If a taxpayer incurs more than one personal casualty loss in a particular year, he must reduce the aggregate loss by 10% AGI.
IV. The $100 floor applies separately to the loss from each single casualty or loss.
V. Basis must be used when determining the value to calculate a casualty loss.

A. I, II
B. II, III, IV, V
C. II, III, IV
D. III, IV, V
E. III, IV

A

C. II, III, IV

The insurance diminishes the loss, but the loss is still deductible with limitations. The value is based on the lower of basis or FMV. The 10% of adjusted gross limitation is applied to all losses in one year.

How well did you know this?
1
Not at all
2
3
4
5
Perfectly
60
Q

For the current tax year, Bob Pearson, a single taxpayer with a $100,000 AGI, has $12,000 of investment interest expense and $8,500 of investment income. He has paid financial advisor fees of $1,500. What amount of investment interest expense, if any, may Bob deduct in the current tax year?

A. $0
B. $8,500
C. $10,000
D. $11,300
E. $12,000

A

B. $8,500

Investment interest expense is deductible up to the amount of that year’s net investment income. Bob’s net investment income is $8,500. The 2% of AGI limitation ($1,500) has been repealed under the TCJA.

How well did you know this?
1
Not at all
2
3
4
5
Perfectly
61
Q

In the current year, Tommy will declare the following income and expenses:

Ordinary dividend income. $8,000
Short-term capital gains. $4,000
Margin interest paid. $19,000
Mortgage interest received. $6,000
Interest rec’d on personal note. $5,000
Credit card interest. $3,000

What amount of margin interest expense can Tommy deduct on Schedule A?

A. $6,000
B. $16,000
C. $18,000
D. $19,000
E. $35,000

A

D. $19,000

Margin interest is only deductible up to investment income [interest ($6,000 + $5,000), ordinary dividends ($8,000), and STCG ($4,000)]. Investment income totals $23,000.

Margin interest paid was $19,000. Interest received is income.

Credit card interest is consumer interest (not deductible) rather than investment (margin) interest paid.

How well did you know this?
1
Not at all
2
3
4
5
Perfectly
62
Q

Mr. Parker, age 57, operates as a sole proprietor. He has no employees (just some 1099 workers). He hopes to retire by age 65.

Which retirement plans would you suggest?

I. Keogh
II. Uni- (k)
III. Defined benefit Keogh
IV. Defined contribution Keogh

A. I, II
B. II, III
С. III, IV

A

B. II, III

With a DB he can aggressively contribute through salary reduction and tax savings in 8 years.

The uni-(k) would also allow him to contribute up to $73,500 (with the catch-up contribution) in 2023. The plan selection depends on how much of his net profit he is willing to contribute.

He has no employees.

Answers I and IV are incomplete answers.

How well did you know this?
1
Not at all
2
3
4
5
Perfectly
63
Q

Bill Richblood owns Richblood, Inc. For years, although the business was profitable, Bill resisted providing a retirement plan. He did not want his business to assume all the paperwork and mandatory employer contributions.

Now the employees are leaving to work for competitors. Bill feels he has to offer some kind of retirement plan to retain key employees. He has consulted various financial planners and he wants to know which one gives the best advice.

Which of the following planner’s advice is the best for Bill?

A. #1 FP - Install a SEP because it can be integrated with Social Security. This means a bigger percentage of the company contribution will be made for Bill. In addition, the plan is simple to install.

B. #2 FP - Install a defined benefit plan because the largest contribution will be made for Bill

С. #3 FP - Install a SIMPLE because of the relatively small mandatory employer contributions

D. #4 FP - Install a cash-less ESOP because the company can contribute stock rather than cash

A

Answer:A

With a SEP there are no mandatory contributions, and it is easy to install.

A defined benefit plan is complicated and can require large, unpredictable mandatory contributions.

An ESOP means that Bill has to share stock with his employees. There is no indication he is willing do so.

There is no indication that the employees would want to contribute to a SIMPLE. Employer contributions are very limited to 3% of compensation.

The SEP will allow for higher employer contribution.

How well did you know this?
1
Not at all
2
3
4
5
Perfectly
64
Q

Gloria Goodtaste owns and operates an upscale gift shop. Revenue varies so much that she cannot hire many full-time employees or provide benefits. The gift shop employs only two full-time workers.

The remainder of her needs are met by several prior employees who only want to work part-time (400 hours per year at most).

What kind of benefits should Gloria offer to the full-time employees that may exclude the part-time employees?

I. Profit-sharing plan
II. SEP
III. Group health insurance
IV. SIMPLE 401 (k)

A. I, III, IV
B. I, III
C. I, IV
D. II, III
E. IV

A

Answer: A

Profit sharing and SIMPLE 401 (k)s are ERISA plans with the 500/1,000 hours per year service eligibility requirement.

Group health insurance plan participation normally requires 30 hours per week.

SEP eligibility requires the 3-year rule for which many of the part-time employees may already qualify.

How well did you know this?
1
Not at all
2
3
4
5
Perfectly
65
Q

Terrific Toys, Inc. reports under the accrual method of accounting. At year end (Christmas) they typically sell substantial amounts of inventory.

The concern is that although they “book” a lot of sales, they may not get the cash until the following year. They have problems funding the company’s profit-sharing plan.

What should the company do if it is reasonably profitable?

A. Change their accounting method from accrual to cash
B. If they cannot fund the plan by their tax filing date, borrow the necessary funds from a bank
C. Wait until next year and pay the corporate taxes due
D. File the corporate tax form showing an amount owed to the profit-sharing plan as a deduction

A

Answer: B

The company cannot change to a cash accounting system.

They can wait another year, but why not save 21% in taxes?

Answer D is not allowed, the cash must be contributed to the plan.

How well did you know this?
1
Not at all
2
3
4
5
Perfectly
66
Q

Success, Inc. has been informed by its plan administrator that its defined benefit plan is overfunded.
The plan administrator indicates that it may be a long time before additional employer contributions can be added to the plan.

What can Success, Inc. do to continue to contribute and deduct new money to the plan?

A. Nothing
B. Select life insurance and annuities to fully fund the plan
C. Continue to make contributions until the IRS sends a notice
D. Find a new plan administrator who will allow them to make new contributions

A

Answer: B

With a fully funded whole life DB plan, the employer can use the actuarial assumptions of the whole life contract.

The actuarial assumptions of the cash value insurance policy are normally lower than most other plan assumptions, allowing for additional contributions.

How well did you know this?
1
Not at all
2
3
4
5
Perfectly
67
Q

Business owners may choose to use qualified plan assets to purchase life insurance for estate planning purposes. (The most available assets to pay the premiums may be in the qualified plan. A popular planning strategy is to purchase a second-to-die insurance policy in a qualified plan.

Which of the following qualified plan (s) can hold second-to-die insurance?

A. Cash-balance plan
B. Profit-sharing plan
C. Money purchase plan
D. All of the above

A

Answer: B

Only profit-sharing plans can hold second-to-die insurance. Pension plans cannot.

Cash balance plans and money purchase plans are pension plans.

How well did you know this?
1
Not at all
2
3
4
5
Perfectly
68
Q

Art Klein just died at age 69. His wife, Patsy, is age 60.

She has worked for the Midwestern Railroad Company for 35 years and will be eligible for railroad retirement benefits of approximately $2,000 per month.

Art had been receiving Social Security retirement benefits of $800 per month.

What benefit is Patsy eligible for now?

A. She is eligible for the Social Security widow’s benefit.
B. She is eligible for Social Security retirement benefits based on her own years of employment.
C. She is eligible for Medicare benefits.
D. She is eligible for the $255 lump sum death benefit.

A

D. She is eligible for the $255 lump sum death benefit.

Surviving spouse railroad employees cannot get a Social Security widow’s benefit and receive railroad retirement benefits. She is not entitled to Medicare until age 65

How well did you know this?
1
Not at all
2
3
4
5
Perfectly
69
Q

Under a 401(k) hardship withdrawal rules, an employee can request an amount equal to which of the following.

I. The entire account balance
II. An amount equal to elective deferrals
III. An amount equal to vested profit sharing contributions
IV. An amount equal to the account earnings

A. I
B. II
C. II, III
D. III, IV

A

C. II, III

For hardship withdrawals, employee’s distributable amount is equal to the employee’s total elective deferrals and vested profit-sharing contributions.

How well did you know this?
1
Not at all
2
3
4
5
Perfectly
70
Q

Stanley participates in an ESOP funded with both company stock and mutual funds. He is concerned about phantom income when she retires and takes distributions from the ESOP. What should you advise him to do to reduce his phantom income exposure?

I. Take only a 72(t) distribution rather than a full distribution in the first year of retirement
II. Take a full distribution of the account balance
III. Take a distribution of the company stock
IV. Roll the whole account into an IRA
V. Roll the mutual funds into an IRA

A. I, IV
B. II
C. III, IV
D. III, V

A

A. I, IV

The IRA rollover of the entire account, which includes the employer stock and the 72(t) distribution eliminate phantom income.

However, the NUA tax break is lost.

How well did you know this?
1
Not at all
2
3
4
5
Perfectly
71
Q

Which of the following is not true about top-heavy plans?

A. A top-heavy plan is one that provides more than 60% of its aggregate accrued benefits or account balances to key employees.
B. For defined contribution plans, employer contributions during a top-heavy year must be at least 3% of compensation.
С. If a defined benefit plan is top-heavy for a given year, it must provide more rapid vesting than generally required.
D. If the plan fails to correct its top-heavy status for 3 years in a row, the DOL can terminate the plan.

A
  1. D - If the plan violates the top-heavy rules, the plan will continue to be qualified but subject to the top-heavy rules (not terminated).
How well did you know this?
1
Not at all
2
3
4
5
Perfectly
72
Q

Karl, age 55, has developed a product that will make him millions over the next years. He has a corporation with two young employees. He is unhealthy and tried to purchase life insurance.
He declined the offer because he was rated and didn’t want to pay the premium. He would like to fund the maximum in a qualified plan to reduce income taxes, benefit his spouse, and have liability protection. Is there anything Karl can do to accomplish all his goals?

A. Install a defined benefit plan ERISA plan
B. Install a 412(e)3 plan [or 412i)] ERISA plan
C. Install a target benefit plan ERISA plan

A
  1. B - He could do a fully funded whole life DB plan. The plan can use the actuarial assumptions of the whole life contracts. These assumptions are normally lower than the plan assumptions (more contributions). The insurance premium will be paid by the plan.
    If at Karl’s death, the difference between the cash surrender value and the face amount is treated as death proceed of life insurance, and is generally excluded from income tax, but only if the insurance cost under Table 2001 (standard rate) has been paid with nondeductible contributions or has been taxable to the employee.
How well did you know this?
1
Not at all
2
3
4
5
Perfectly
73
Q

Sue’s employer has a profit-sharing plan. The employer has not made a contribution for the prior year. In addition, there was no employee turnover. Sue’s salary was $85,000. Her W-2 shows that she wasn’t an active participant. Can she deduct her IRA contribution for last year?

A.Yes
B. No
C. Maybe

A
  1. A - Yes, if there were no annual additions to the account for the prior year (contributions or forfeitures), she could fund a deductible IRA. Sue’s W-2 would show she wasn’t a participant.
How well did you know this?
1
Not at all
2
3
4
5
Perfectly
74
Q

It is December 24th. The day has been full of phone calls and appointments. You are exhausted.

You are about to go on vacation when a smartly dressed lady, Kate, walks into your office. She was referred to you by your wealthiest client.

Kate’s husband died this year and she inherited his $2 million IRA. She wants to make a full Roth conversion this afternoon.

What is the most important information that you need before assisting Kate with her conversion?

A. Her age
B. Her AGI
С. Will she remarry?
D. Her CPA’s name

A
  1. A - Kate can’t convert the whole IRA to a Roth without taking a RMD if she is 73 or older. The first dollars withdrawn from the IRA are deemed to be the RMD until that amount is distributed. Once the RMD is withdrawn, then the remaining balance can be converted. Her age is important. (Pre-study)
How well did you know this?
1
Not at all
2
3
4
5
Perfectly
75
Q

Question 14
Cynthia’s final-average monthly salary is $4,800. To arrive at her benefit, multiply 1.25% by the $4,800 final-average monthly salary by 25 (the number of years of service). Cynthia’s monthly retirement benefit will be equal to $1,500 paid in the form of a life annuity. This is an example of what type of defined benefit formula?

A. Flat-percentage-of-earnings
B. Flat-amount-per-year of service
C. Unit-benefit
D. Flat-benefit-percentage

A

Question 14: C

Answer C is also known as percentage-of-earnings-per-year of service. Answer D is nonsense.

How well did you know this?
1
Not at all
2
3
4
5
Perfectly
76
Q

Question 11
In which of the following qualified retirement plans may forfeitures be allocated to increase account balances of the remaining plan participants?

I. Defined benefit plan
II. Profit-sharing plan
Ill. Money purchase plan
IV. Cash balance plan

A. I, II
B. I, IV
C. II, III
D. II, IV

A

Question 11: C

Forfeitures in defined benefit plans and cash balance plans must reduce plan costs or contributions. Money purchase plan forfeitures may (not must) be allocated to employee account balances. Forfeitures in a profit-sharing plan normally are allocated to the plan participants.

How well did you know this?
1
Not at all
2
3
4
5
Perfectly
77
Q

Question 9
On what is the maximum deductible contribution in a target benefit plan based?

A. An actuarial determination
B. The minimum-participation rule
C. A maximum of 25% of the aggregate eligible compensation of all covered participants
D. A maximum of 25% of the firm’s total payroll

A

Question 9: C
A target benefit plan is a defined contribution plan. Although an actuarial calculation is made when a target benefit plan is first installed, the maximum deductible contribution is always limited by Answer C. This is true even if the actuarial calculation calls for a larger contribution (likely if there are many older highly compensated employees and relatively few lower-paid rank-and-file employees). Answer D is wrong. The contribution is not based on total payroll but eligible compensation or payroll.

How well did you know this?
1
Not at all
2
3
4
5
Perfectly
78
Q

Question 5
What impact would increasing investment returns have on a money purchase plan?
A. Employer contributions would increase
B. Employer contributions would decrease
C. The amount of the employer’s contribution would not change

A

Question 5: C

Investment returns affect account balances, not contributions.

How well did you know this?
1
Not at all
2
3
4
5
Perfectly
79
Q

Question 4
What would be the impact on a money purchase plan if a key employee retires and is replaced by a clerical employee?

A. Company contributions would increase
B. Company contributions would decrease
C. The amount of the company contribution would not change

A

Question 4: B

The key employee had a higher salary level than the clerical employee. The employer would be contributing less on behalf of a lower paid worker.

How well did you know this?
1
Not at all
2
3
4
5
Perfectly
80
Q

Question 1
Quick Manufacturing, Inc. always maintains a top-heavy profit-sharing plan because of employee turnover due to layoffs. Which vesting schedule should they adopt if the company feels this will be an ongoing situation?

A. 3-year cliff
B. 5-year cliff
C. 2- to 6-year graded
D. 100% with 2-year eligibility

A

Question 1: A

The employees will be eligible after one year, but unless they stay three years, the employees will forfeit all employer contributions made to the plan. The forfeitures will be allocated to the long-term employees (probably the HCEs) allowing them to potentially receive 2023 annual additions of up to 100% of compensation or $66,000.

There is no indication the company wants to retain employees (2-6 year graded).

The 100% vested plan (2-year eligibility) is not an advantage over the 3-year cliff vesting schedule.

How well did you know this?
1
Not at all
2
3
4
5
Perfectly
81
Q

Question 7
Together, four doctors own 100% of Labs, Inc., a support organization. The respective ownership percentages are 80%, 10%, 5%, and 5%. The doctors own and operate individual practices as personal service corporations with no employees. Each practice provides a defined benefit plan. Which type of controlled group would Labs, Inc. fall under?

A. Parent-subsidiary controlled group (80% combined)
B. Brother-sister controlled group (80% combined)
C. Affiliated service group
D. Leased employees

A

Question 7: C

The IRS defines this as an affiliate service group. I realize the 80% factor may lead you to answer this as brother-sister.

How well did you know this?
1
Not at all
2
3
4
5
Perfectly
82
Q

Question 2
Which of the following is true about plan loans from a 401(k) plan?

A. They are prohibited because the plan accepts employee elective deferrals.
B. The loan interest is never deductible.
C. If married, the spouse would not need to consent to the loan.
D. The loan does not have to be secured.
E. If a plan allows loans, they must be made available to all participants without discrimination.

A

Question 2: E
Interest paid on a plan loan for a principal residence may be deductible. If the plan is subject to QPSA/QJSA and if the loan is secured with plan assets, both spouses must consent to the loan. In all cases, the loans must be adequately secured (with plan assets or other collateral acceptable to the plan administrator).

How well did you know this?
1
Not at all
2
3
4
5
Perfectly
83
Q

Question 4
Walter Workaholic works for two related employers. Each employer provides a 401(k) plan. With one employer he earns $50,000, and with the second employer he earns $60,000.

If both plans allow for a 6% deferral and a 3% match, how much can he defer in 2023?

A. $3,000
B. $3,300
C. $6,600
D. $22,500
E. $30,000

A

Question 4: C

6% of the total of $110,000 = $6,600. The plan only allows for a 6% deferral. He cannot defer $22,500.

How well did you know this?
1
Not at all
2
3
4
5
Perfectly
84
Q

Question 5
A plan participant borrows from her 401(k) account to purchase a house. Under which of the circumstances described below would the loan interest be deductible?

A. The participant, a key employee, secures the loan with the primary residence purchased with the loan.

В. Under the tax code, home mortgage interest is deductible.

C. The participant, a rank-and-file employee, secures the loan with both the primary residence purchased with the loan and elective deferrals.

D. The participant, a rank-and-file employee, secures the loan only with the primary residence purchased with the loan.

A

Question 5: D

401(k)s [and 403(b)s] plans do allow principal residence loans to key employees and non-key employees alike.

However, interest paid on a plan loan for a principal residence is only deductible under two conditions.
1) The loan is secured by the residence for which the loan is made, and
2) The participant is not a key employee.
Interest on plan loans (even if for a primary residence) will be considered consumer interest if secured by the employee’s own elective deferrals. Home mortgage interest isn’t always deductible. Consumer interest is not deductible.

How well did you know this?
1
Not at all
2
3
4
5
Perfectly
85
Q

Question 6
Under which of the following circumstances can a Roth account be closed with no adverse tax consequences?

A. Immediately after it is funded
B. When the distribution is made for a qualifying special purpose
C. After the individual has maintained the Roth IRA for five years D. After the owner turns age 59½
E. Both C and D

A

Question 6: E
Earnings must meet the five-year rule, and the Roth IRA owner must be 59½ for the distribution to be tax-free.

How well did you know this?
1
Not at all
2
3
4
5
Perfectly
86
Q

Question 3
Tom Sellers own and operates TS, Inc. TS, Inc. has no other employees. Income from the business net of expenses is $80,000 this year.

Which of the following plans provides the maximum allowable contribution for Tom?

A. SEP
B. SIMPLE 401(k)
C. SIMPLE IRA
D. Keogh

A

Question 3: A
A SEP is easy to install. The SEP contribution would be $80,000 x .25 = $20,000. SIMPLE IRA and SIMPLE 401(k)
contributions are $15,500 + $2,400 (3%) = $17,900. Keogh isn’t a complete answer. Is the Keogh a defined
benefit or money purchase plan?

How well did you know this?
1
Not at all
2
3
4
5
Perfectly
87
Q

Question 13
Mr. Pope, age 55, operates his management consulting business as a sole proprietor. He wants to establish an uncomplicated retirement plan and contribute the maximum allowable amount per year. He has no employees, and his annual net profit always exceeds $250,000. Which type of plan should he adopt?

A. SIMPLE 401(k)
B. Profit-sharing
C. SIMPLE
D. SEP
E. uni-401(k)

A

Question 13: E

With the uni-401(k) Mr. Pope can defer $22,500 plus add employer profit-sharing contributions to a cap of $66,000 plus a catch-up contribution of $7,500.

The SIMPLE and SIMPLE 401(k) would only allow for $15,500 to be deferred plus 3% of compensation as a matching contribution.

The profit-sharing and SEP would allow Mr. Pope to contribute $66,000 but no catch-up contribution is available.

How well did you know this?
1
Not at all
2
3
4
5
Perfectly
88
Q

Question 1
A defined benefit plan includes incidental life insurance using the 100 to 1 test. Tom, who had been a participant, died prior to retiring. His wife received two checks during the year of his death: $200,000 pure death benefit and $20,000 cash value. What amount will not be taxable this year?

A. 0
B. $20,000
C. $200,000
D. $220,000

A

Question 1: C

The pure life insurance death benefit will be tax-free. The employee was charged with the cost of the pure death benefit of the life insurance. The cash value will be taxable. See Chapter 8 materials for more detailed explanation.

How well did you know this?
1
Not at all
2
3
4
5
Perfectly
89
Q

Question 3
XYZ’s pension plan portfolio consists of the following investments. Which investments) may produce UBTI income?

A. Real property rents
B. Gain from sale of capital assets
C. Equipment leasing program (containers)
D. Annuities (fixed or variable)
E. Whole life insurance

A

Question 3: C

Equipment leasing programs are normally limited partnerships. Annual limited partnership income in excess of $1,000 is exposed to UBTI current taxation.

How well did you know this?
1
Not at all
2
3
4
5
Perfectly
90
Q

Question 4
What entity or regulation imposes extensive reporting and disclosure requirements on a defined benefit plan?

A. PBGC
B. ERISA
C. Department of Labor
D. IRS

A

Question 4: B

A defined benefit plan is subject to all the ERISA requirements for qualified plans (participation, funding, vesting, etc.) and the ERISA reporting and disclosure requirements. This information is disclosed to the plan participants and/or filed with the IRS or the Department of Labor. PBGC is not per se a reporting agency.

How well did you know this?
1
Not at all
2
3
4
5
Perfectly
91
Q

Question 5
Apex, Inc. wants to reward its employees but does not have cash to contribute for year-end. The company feels it will be in a very profitable position during the year 2023. What would you suggest Apex do?

A. Adopt a profit-sharing plan and, in lieu of a cash contribution, provide the plan with a promissory note.
B. Adopt a profit-sharing plan and borrow the necessary cash for the contributions from a bank.
C. Adopt an ESOP and fund the contribution with company stock.
D. Do not start the plan until 2024.

A

Question 5: B

This option allows Apex to put money in the plan now and get a tax deduction now. Answer C is a good answer, but there is no indication in the question that Apex is interested in using company stock. This is the best answer though answer D could work as well (subjective).

How well did you know this?
1
Not at all
2
3
4
5
Perfectly
92
Q

Question 11
Joe Mills dies a few months after his 69th birthday. His wife, Wendy, is the sole beneficiary of Joe’s IRA. Match his IRA to the correct distributions.

I. Take distributions at Joe’s RBD based on Wendy’s single life expectancy recalculated each year

II. Roll the account balance into Wendy’s IRA; take distributions based on her RBD (new uniform lifetime
table)

III. Take distributions over Wendy’s life expectancy by December 31st of the year after Joe died

IV. Take distributions at least as rapidly as under the distribution schedule in effect when Joe died

A. I
B. I, II
C. III
D. II, IV

A

Question 11: B

Answer III would be more appropriate for a non-spouse beneficiary. Joe had not started his distributions so the
“at least as rapidly” distribution would not apply in this situation.

How well did you know this?
1
Not at all
2
3
4
5
Perfectly
93
Q

Which of the following statements accurately describe(s) the provisions of constructive receipt as it is applied to nonqualified deferred compensation plans?

I. Constructive receipt occurs when the funds are available to the employee.

II. Constructive receipt by employee results in taxation to the employee of the applicable benefits.

III. If a company goes through a merger or acquisition, the rabbi trust provisions will automatically trigger constructive receipt to the employee.

IV. If a company owns the deferred compensation assets, its employee will not have constructive receipt.

A. I, II, III, IV
B. I, II, IV
C. I, II
D. III
E. IV

A

Question 1: B
A rabbi trust might trigger constructive receipt due to a merger or acquisition.

How well did you know this?
1
Not at all
2
3
4
5
Perfectly
94
Q

Question 3
John, a high-performing sales manager for ABC Auto Parts, is unhappy with the company’s 401(k) program. The $330,000 compensation cap and the ADP test are limiting contributions.

In an effort to retain John, which of the following opportunities should ABC make available to him?

A. A salary continuation plan invested in a variable annuity policy
B. An increase in company contributions to the 401(k)
C. A secular trust
D. A split-dollar policy
E. A pure deferred compensation arrangement using a VUL policy

A

Question 3: A

A salary continuation plan is funded entirely by employer contributions. Pure deferred compensation uses a portion of the employee’s current compensation. Nothing indicates a need for life insurance (Answers D and E).

A secular trust contribution would be currently taxable to John and limits his ability to use the funds until the end of the term of the agreement.

How well did you know this?
1
Not at all
2
3
4
5
Perfectly
95
Q

Question 4
Harry is granted $250,000 of ISO options that vest in one year. The following year he exercises $150,000 of the options. What will be the result of this exercise?

A. $100,000 will be treated as ISOs; $50,000 will be treated as NSOs.

B. $50,000 will be treated as ISOs; $100,000 will be treated as NSOs.

C. $75,000 will be treated as ISOs; $75,000 will be treated as NSOs.

D. $150,000 will be treated as NSOs.

A

Question 4: A

If more than $100,000 of ISOs that vest in the same year are granted, only the first $100,000 (worth) are treated as ISOs with the excess treated as NSOs

How well did you know this?
1
Not at all
2
3
4
5
Perfectly
96
Q

Can the employer that maintains a profit-sharing plan contribute more than 25% to an employee’s account?

A. No, because the limit is 25%
B. Yes, because the limit is the lesser of 100% or $66,000
C. Yes, as long as the total company contributions do not exceed 25% of total plan compensation, the employer can contribute more than 25%.
D. Both B and C

A

Question 4: D
Individual employees can receive contributions in excess of 25% providing the contributions conform to the 415 limit (as long as company contributions do not exceed 25% of total eligible plan compensation).

97
Q

Question 13
Debra Snow, age 50, works for Tilden, Inc. Tilden provides a defined benefit pension plan. Debra is the VP of creative marketing. She earns $200,000 per year from Tilden. Debra is also a talented artist. This year she sold paintings netting $100,000 after expenses this year. If Tilden adds $100,000 to her defined benefit plan this year, can Debra also fund a SEP?

A. No, the $100,000 exceeds the 415 limit of 25% or $66,000.
B. Yes, but it is limited to 25% of $100,000 less Social Security offset or $18,590 (18.59%)
C. Yes, but it is limited to $22,500.
D. No, she is already a participant in a DB plan, and the 1.0 rule applies.

A

Question 13: B

SEP contribution rules are similar to those for a Keogh PS Plan. For self-employed individuals, the maximum contribution percentage is 18.59% (Note: $18,590/$100,000 is 18.59%) of earned income with a 25% plan, and for corporate employees (including owners) the contribution limit is 25%. The 1.0 rule has been repealed.

Note: She works for two unrelated employers, so both contributions are allowed.

98
Q

Question 14
Mary Jane’s husband died this year. He participated in a qualified plan at work. Mary Jane, age 53, also participates in a qualified plan at work. She doesn’t need his qualified plan distribution now or in the immediate future. Since her husband’s death, May Jane is uncertain whether she will retire early or not. She would like to roll the qualified plan over into her name. Which choice below is a workable alternative for Mary Jane?

A. Make a direct rollover into an IRA account in her name.
B. Make a direct rollover into her qualified plan at work.
C. Take a direct distribution and open an IRA account in her name.
D. Leave the funds in her deceased husband’s qualified plan and change the name on the account to her name.

A

Question 14: B

If she makes a direct rollover into her IRA, she will be subject to the IRA rules. If she makes a direct rollover into her own employer’s qualified plan, she can use separation from service at age 55 (no 10% penalty) or wait until after retirement to take RMDs. This gives her greater flexibility considering her uncertainty as to when she will actually retire, plus with a qualified plan, the assets are fully creditor protected. See Chapter 9 for rationale.

99
Q

Question 19
Mr. Hale wants to borrow money to purchase additional investments. He would like to deduct the loan interest.
Which of the following assets makes the most sense to pledge as collateral for a margin (account) loan?

A. $100,000 of municipal bonds
B. $100,000 IRA account
C. $100,000 of common stock
D. $100,000 in a money market account

A

Question 19: C

Pledging the stock is like buying stock on the margin. It makes the most sense. The loan interest deduction is disallowed when municipal bonds are pledged for a loan. Municipal income is not investment income. If an IRA is pledged for a loan, the IRA will be disqualified.

100
Q

Question 28
Should Bill change the type of profit-sharing plan that his practice provides? If so, why?

A. No, he should leave the plan alone.
B. Yes, he should change to a tandem plan to be guaranteed at least a 25% contribution each year.
C. Yes, he should change to a defined benefit plan to get the maximum benefits to retire in 10 years.
D. Yes, he should discontinue any kind of pension or profit-sharing plan and adopt a SEP.

A

Question 28: A

No, Bill barely can pay himself enough to survive. Additional contributions (such as to a money purchase, defined benefit plan, or SEP) would have to come from his personal income.

101
Q

Question 29

What is Bill’s AGI for the current year?
Read the footnotes along with case data carefully, and remember the P.A. operates as an S corporation. He is married filing separately (affects other questions).
NOTE: The CFP Board exam will not give you these types of hints.

A. $202,500
B. $202,800
C. $214,000
D. $225,300
E. $229,500

A

Question 29: A

$250,000 - Bill’s income ^ ^ ^
-$1,500 - less capital losses ^
-$36,000 - less alimony
-$4,000 - less alimony ^ ^
-$6,000 - less 100% health premium ^ ^ ($500x2)
———————-
$202,500 - AGI

^ Married filing separately allows only ½ ($1,500) on capital losses.

^ ^ The life policy is paid separately from the monthly alimony ($4,000). The P.A. cannot deduct Bill and Martha’s health premium (It becomes part of his income - S corporation rules). He can personally deduct 100%.
The $3,000 of dividends and the $7,500 of capital gains come from her mutual fund (see question 46).

^ ^ ^ This includes disability and medical insurance payments— see footnote 1 of the financial statement.

102
Q

Question 35
Bill is concerned about his low level of malpractice coverage. Considering the facts of the case, which of the following choices benefit Bill?

I. Invest available future earnings into a variable annuity

II. Each year take all profits out of the business as salary

Ill. Raise the deductible and increase the coverage as much as possible

IV. Use future earnings to purchase cash value life insurance

A. All of the above
B. I, II
C. I, III, IV
D. II, III, IV
E. I

A

Question 35: A

The variable annuity is creditor protected. Removing assets from the P.A. makes the business less valuable. Bill could take more salary now and invest in annuities or life insurance. He can improve his malpractice per Answer IlI. Answers Il and V strip all the value out of his P.A. (PSC Corporation). He can then put the cash into an annuity or cash value life insurance policy. Go back to General Principles 10-6 for additional information.

103
Q

Question 39
What can Bill do to motivate his son to join him in his medical practice?

A. Provide him with occasional sporting event tickets
B. Shorten the eligibility and vesting requirements for the profit-sharing 401(k)
C. Pay off all the college loans
D. Provide him with a company car
E. Pay him a guaranteed salary plus bonus

A

Question 39: E

As a young, new medical practitioner, Richard will probably be more attracted by cash than by longer-term benefits. Changing the eligibility and vesting requirements for the 401(k) might not impress his young son.
There is no indication that his son has student loans to pay off. Bill is in poor health working alone. He needs his son to help him before he kills himself.

104
Q

Question 41
Dan and Jane estimate that they need $3,200,000 (to be safe) and that their current assets will grow to a value of $1,900,000 as of the first year of retirement.

They will need to save $50,000+ per year to meet their goal.

Identify the qualified retirement plan that would be most appropriate to install in Dan’s company.

A. Money purchase plan
B. Target benefit plan
C. Profit-sharing 401(k)
D. LESOP
E. Tandem plan (money purchase and profit-sharing combined)

A

Question 41: C

The allowable contribution for Bill into a 401(k) plan with a catch-up contribution would be $73,500 for 2023 ($66,000 + $7,500). This seems affordable and allows him to defer some income. The money purchase and target plans don’t permit catch-up contributions. A tandem plan was used before 2001 to create 25% contribution. 15% was the maximum for profit sharing prior to 2001, so a money purchase plan had to be added (10%).

Note: He may want to take more salary to make answers A, B, or C work (Rule 404 - 25%).

105
Q

Question 44
Assume that Dan establishes a qualified profit-sharing plan for Sounds Terrific, Inc. allowing for employer and employee contributions, plan loans, and in-service withdrawals (The plan requires 10 years of service for in-service withdrawals). Further assume that Dan will be a participant from 2017 until his distributions from the plan begin. Dan has certain concerns regarding the future distributions he will receive from the plan.
Assume that when Dan is age 58, his retirement plan account is valued at $200,000 and then answer 44 and 45.
To fund the purchase of a new home,

Dan withdraws 20% of his account balance in the plan. What penalty tax and income tax, if any, are imposed in this situation?

A. None, as this is less than $50,000 and would be considered a hardship withdrawal
B. Ordinary income tax only
C. 10% early withdrawal penalty and ordinary income tax
D. No penalty or income tax if he followed plan guidelines for withdrawals
E. If Dan repaid the distribution to the plan within 60 days, there would be no penalty or income tax

A

Question 44: C

He cannot qualify for a hardship withdrawal. He is 58. He hasn’t met the 10 year of reserves for in-service withdrawals.

106
Q

Question:
Identify the open-ended question from the list below.

Response:
A. At what age would you like to retire?
B. Who is the executor of your estate?
C. Can you tell me more about your experience in covered call writing?
D. Are you satisfied with your tax preparer?

A

C. Can you tell me more about your experience in covered call writing?

Asking the client to explain more about an experience is an open-ended question. Although asking the client when he or she would like to retire is a question seeking a qualitative answer it is still a closed-end question. The name of the executor is a closed-end question seeking a quantitative fact (a name). Asking whether or not someone is satisfied with another professional is most likely to produce a “yes”/’no” response.

107
Q

Question:
Client A is insistent on keeping the stock of company Q because her grandfather gave it to her as a child. In addition, the stock is not doing well and does not fit into the client’s overall investment allocation. What type of behavioral bias is the client exhibiting?

Response:
A. Cognitive bias
B. Emotional bias
C. Irrational bias
D. Endowment bias

A

D. Endowment bias

Endowment bias is a type of bias that causes individuals to value an owned object (such as a stock or piece of real estate) at a higher value than its market value. A, B and C are all possible answers, but endowment bias is clearly being exhibited in this question.

108
Q
  1. Harry Porter started working immediately after he graduated from college. He earned an engineering degree specializing in computer software. At his first job, he immediately started making $39,000 annually. Since then, every year he has been making 401(k) maximum deferrals plus maximum Roth IRA contributions. This year he contributed $6,000. His total contributions to date are $42,500 and his Roth IRA has a balance of $62,500. Now Harry has an opportunity to buy his first house that is a few minutes away from his job. He needs $52,500 as a down payment to get a very attractive mortgage rate. If he withdraws $52,500 what will be his tax situation?

A. The $52,500 will be income tax free and no 10% early withdrawal penalty would apply.
B. $42,500 will be income tax free and $10,000 will be subject to income tax plus a 10% penalty
C. $42,500 will be income tax free and $10,000 will be subject to income tax
D. $37,500 will be income tax free and $15,500 will be subject to income tax. The $5,500 contribution made this year is included.

A

A. The $52,500 will be income tax free and no 10% early withdrawal penalty would apply.

Harry’s regular Roth IRA contributions are distributed first and are tax-free. Additionally, he qualified under the five-year holding period to withdraw $10,000 for a first home purchase. The $10,000 is tax-free and no penalty applies.

109
Q
  1. Mac Blair decided to make a gift of Blair, Inc. common stock to his son Blake. Mac seeks that any future appreciation of the stock not be included in his estate for federal estate tax purposes. He is going to retire soon and will need income during his post-retirement years. Mac has converted the majority of his common stock to preferred stock and gifted the remaining common stock to his son. What is the result?
    Choose one

A. The value of the common stock for gift tax purposes will be based on dividends paid on the preferred stock.
B. The value of the preferred stock for gift tax purposes will be based on the FMV of the business less the value of the common stock.
C. The value of the common stock for gift tax purposes will be based on the total FMV of the business.
D. The value of the common stock for federal gift tax purposes will be based on a business appraisal performed by a qualified appraiser.

A

C. The value of the common stock for gift tax purposes will be based on the total FMV of the business.

The preferred share value will be determined based on the stated dividends. The common stock value would then be the difference between the FMV of the corporation and the aggregate value of the preferred shares.

110
Q

Question:
2. Ted pays alimony from a 2018 divorce to his ex-wife, Cruella, in the following amounts. Has Ted made an “excess alimony payment”?
• Year 1 $10,000
• Year 2 $30,000
• Year 3 $10,000
• Year 4 $10,000

Response:
A. Yes, in the first post-separation year
B. Yes, in the second post-separation year
C. Yes, in the third post-separation year
D. Yes, in the fourth post-separation year
E. No, the alimony is not “front-loaded”

A

B. Yes, in the second post-separation year

To determine excess alimony, note the difference in the payment between years 2 and 3. Under the Internal Revenue Code, the excess alimony payment for the second year is the amount that exceeds the payment in the third year by more than $15,000. The excess is $5,000; [$30,000 - $10,000 + 15,000)]. Doubtful this will be tested.

111
Q

Question:
6. Which of the following is true about net present value (NPV)?

A. The investment should generally be made if the NPV is negative.
B. The investment should generally be made if the NPV is positive.
C. Its calculation discounts unequal cash flows at a required rate of return less the initial cost of an investment.
D. Its calculation can only include positive cash flow entries.

A

Feedback:
Answer C is the definition of NPV. NPV, while not a perfect valuation tool provides the planner with a reasonably reliable model from which to make assessments and projections. The calculation should factor both positive and negative entries.

112
Q
  1. Patrick Parker died at age 44 with most of his property in joint tenancy (JTWROS) or with a beneficiary designation. If Mrs. Parker (who is wealthy) does not want to receive all of the joint tenancy property, what amount of property do you recommend that she disclaim?

Response:
A. None of the property. It would produce a better tax outcome if she claims portability of the unused estate tax exemption from Patrick’s estate rather than disclaim the joint tenancy property.
B. Up to the exemption amount of $12,920,000
C. Up to any amount with which she feels comfortable
D. All the joint tenancy property that was attributable to Patrick

A

C. Up to any amount with which she feels comfortable

Feedback:
Mrs. Parker does not need Patrick’s assets. The portability will only allow $12,920,000 to pass tax-free from Patrick’s estate. What happens if his assets triple by the time she dies.

113
Q
  1. Your client, Roger, is looking for an investment that will accomplish his objective of income tax deferral. Which of the investment vehicles shown below would defer income taxes?
    Response:

A. A rental apartment building
B. A municipal bond
C. A Single premium annuity
D. A Certificate of deposit

A

A. A rental apartment building

Feedback:
Investing in the rental apartment building enables depreciation deduction to offset rental income on an asset that can appreciate in value over time. The single premium annuity shown does not indicate if it is immediate or deferred. Municipal bond interest is tax exempt rather than tax deferred. Interest on Certificates of Deposit is currently taxable.

D. A Certificate of deposit

114
Q
  1. Spike has engaged you because his next door neighbor praised your services as a financial planner. That said, Spike does not appear to connect with the financial planning process. He does not look you in the eye and argues with every recommendation you offer. Further, he makes comments like, “You’re just trying to make money off me” and “ You do not know what you are talking about”. You are an experienced financial planner and a Registered Investment Advisor. Under the CFP® Board Code of Ethics which statement below is most supportable?

Response:
A. You cannot terminate the professional relationship with Spike because assigning a client relationship is prohibited.
B. You can terminate the relationship with Spike if he makes you uncomfortable.
C. You can transfer Spike’s account to another advisor who is more patient.
D. You should suggest alternative financial planning strategies to Spike.

A

B. You can terminate the relationship with Spike if he makes you uncomfortable.

Feedback:
Under the CFP® Board Code of Ethics, the scope of the client relationship must be mutually acceptable to both client and planner. When it is no longer so either party can terminate the relationship. While the 1940 Investment Advisors Act does prohibit assignment of the relationship (Answer A) terminating the relationship is not assigning it to another. As an ethical planner, you have already made your best recommendations to Spike.

115
Q
  1. Mrs. Tillman, age 58, has an assortment of health problems such as chronic back pain, obesity, high blood pressure, etc. Mr. Tillman, age 65, is retired after working for one employer for 40+ years, but he is in good health. The Tillmans own about $1.6 million in assets. Their cash flow meets their expenses sometimes with a little to spare. If Mr. Tillman would consider only one of the types of insurance contracts shown below, which would be your strongest recommendation?

Response:
A. Long-term care insurance/Life Insurance with LTC rider
B. Life insurance
C. A Variable annuity
D. Medical expense insurance
some of her IRA assets.

A

A. Long-term care insurance/Life Insurance with LTC rider

Feedback:
If Mrs. Tillman needs long-term care, Mr. Tillman may have to use their assets for the expenses relating to her care. He needs to protect their assets in case he needs long-term care.
The question asks about insurance for Mr. Tillman rather than for Mrs. Tillman. He is 65, and after working for 40+ years should qualify for Medicare.

116
Q
  1. Mr. Boyd has been your client for many years. Mr. Boyd’s portfolio has weathered ups and down cycles in the economy. His average return over time is approximately 11 percent. Now Mr. Boyd wants to buy a $35,000 car. His old car has very limited trade in value. He asked you whether he should buy the car for cash, which he has available, or lease the car under a 3-year lease agreement. How would you best respond?

Response:
A. “Mr. Boyd, under a lease arrangement, after 3 years of payments you will have no trade in value.”
B. “Mr. Boyd, you should consider the opportunity cost of investing the money and lease the car.”
C. “Mr. Boyd, since you plan to keep the car for more than 3-years, you should purchase the car.”
D. “Mr. Boyd, because lease agreements have numerous restrictions, you should purchase the car.”

A

B. “Mr. Boyd, you should consider the opportunity cost of investing the money and lease the car.”

Feedback:
The decision as to whether it is best to buy versus lease a car is often based on opportunity cost. First, determine the break even return on the investment. If investing can outperform the breakeven return, it makes sense to the car.

117
Q
  1. Alice and Leo Fuller (both age 30) have come to you. Alice earns $60,000/year working full time as an employment counselor. Leo earns $50,000/year working full time as a retail sales manager. Now Alice wants to have children and then work only part/half-time. The Fullers have accumulated $300,000 in savings in short-term fixed income investments. Their three-bedroom home has a current FMV of $225,000. The current mortgage balance is $150,000 and the note carries a 4% fixed interest rate. The Fullers want to retire at age 55. Given their objectives, how should the Fullers proceed?

Response:
A. They should sell the home and rent. If Alice works only part-time and they have children, the Fullers cannot afford the house.
B. They should keep the home. Even if Alice works only part-time, the Fullers can still afford to keep the home and have room for children.
C. The Fullers should use some of their savings to pay down the mortgage to a level where the PITI can be covered under their reduced income.
D. Given their objectives, renting will give the Fullers the flexibility they need.

A

B. They should keep the home. Even if Alice works only part-time, the Fullers can still afford to keep the home and have room for children.

Feedback:
Presuming that the Fullers have children, they may need the bigger house. The home is usually an effective inflation hedge. One generally can’t pay down a mortgage to reduce PITI.

PITI remains relatively constant even as the mortgage is paid down. The loan will be paid off faster, but the mortgage agreement (PITI) would remain relatively unchanged. The $225,000 home does not seem extravagant.

118
Q

Question:
51. Sam believes he was cheated on an investment transaction with a broker/dealer with which you are not affiliated. As a financial planner, if Sam is seeking recourse, how would you recommend that he proceed first?
Response:
A. Sam should notify the CFP® Board
B. Tell Sam to hire an attorney and sue the broker/dealer that handled the transaction
C. Tell Sam to report the transaction to the FINA and SEC
D. Take the broker/dealer to arbitration through the FINRA procedures
E. Write to the compliance officer (registered principal) of the broker/dealer that handled the transaction and ask for his money back

A

E. Write to the compliance officer (registered principal) of the broker/dealer that handled the transaction and ask for his money back

Feedback:
A disgruntled client of a broker/dealer should generally begin the complaint process through that broker/dealer’s compliance function. If the firm is unresponsive or disagrees with the customer’s claim, arbitration should be pursued through FINRA. Mediation may also be available. the broker/dealer community is governed by FINRA directly under the supervision of the SEC. The CFP Board is not the regulator for broker/dealers.

119
Q
  1. An individual holding a FINRA Series 6 license can receive commissions relative to the sales of which of the following investments?

I. Mutual funds
Il. Variable life insurance
Ill. Variable annuities
IV. ETFs

Response:
A. I, II, III, IV
B. I, II, III
C. I, IV
D. II, III
E. I only

A

E. I only

A Series 6 limited securities representative licensed individual can sell mutual funds only. To sell variable products, the individual must also hold a state-issued insurance producer’s license. The question does not indicate any insurance license. ETFs are Exchange traded funds. They are normally traded on a national exchange without a prospectus. To earn commission for sales of products not accompanied by a prospectus, one must hold the FINRA Series 7 general securities

120
Q
  1. Sonia Appletree owns an upscale retail gift shop. Business varies so much that she cannot hire full-time employees or provide benefits. She has decided to limit the number of full-time workers to two employees. The remainder of her employee needs can be met by prior employees who only want to work part-time, typically 400 hours per year at $10 per hour. What kind of benefits would you suggest to Sonia that would cover herself and the full-time employees and would exclude the part-time employees?
    I. A profit-sharing plan
    II. A SEP
    III. A group health insurance plan
    IV. A SIMPLE 401(k)

Response:
A. I, Il
B. I, III
C.I, IV
D. II, III
E. III, IV

A

B. I, III

Feedback:
Under ERISA rules, a profit sharing plan may exclude from participation employees working fewer than 1,000 hours per year. Eligibility to participate in an employer-provided group health insurance plan normally requires 32 hours per week. SEP eligibility falls under the 3-year rule which could force Sonia to cover certain returning part-time workers. The SIMPLE (401k) implies employee deferrals. At this point it is not clear as to whether the employees would want to make elective deferrals. Her own maximum contribution would be lower than that under the profit-sharing plan.

121
Q
  1. In 2016, Sidney and Ruth Silverman borrowed $100,000 from a bank pledging their home as collateral. At the time of the loan, the Silverman’s equity in their home was $125,000. The proceeds from the loan were used to start their new delicatessen business, Crosby, Stills, and Nosh. They also used other funds to fully equip the business. Which statement below best describes the tax result of this home equity loan in tax year 2023?

Response:
A. The interest on the home equity loan is fully deductible because the amount borrowed did not exceed the Silverman’s equity in the home used to collateralize the loan.

B. The interest on the home equity loan is fully deductible and reported as an itemized deduction on Schedule A because the loan agreement was executed before the enactment of the Tax Cuts and Jobs Act of 2017.

C. The interest on the home equity loan is not deductible because miscellaneous itemized deductions have been repealed under the Tax Cuts and Jobs Act of 2017.

D. The interest on the home equity loan is not deductible because interest on home equity loans is generally nondeductible in tax year 2023.

A

D. The interest on the home equity loan is not deductible because interest on home equity loans is generally nondeductible in tax year 2023.

Feedback:
Interest on home equity loans is generally nondeductible unless utilized for home improvement. While interest on a home equity loan was an itemized deduction, it was not classified as a “miscellaneous itemized deduction.” However, interest on home mortgages up to $1M are grandfathered if taken out prior to December 15, 2017.

122
Q
  1. Sheldon Shrink, MD is a practicing psychiatrist. He operates his sole practice as an LLC. After covering expenses such as office space and malpractice insurance, Dr. Shrink’s practice has net revenue of $700,000. Sheldon is married to Shirley Shrink, who is an architect working for a major urban design firm. Her annual salary is $200,000. The Shrinks file their taxes jointly reporting AGI of $900,000 and taxable income of $827,000. To what amount, if any, of qualified business income (QBI) are the Shrinks entitled on their 2023 form 1040?

Response:
A. $0
B. $40,000
C. $140,000
D. $180,000

A

A. $0

The psychiatric practice is a “service related” business. The owner of a service business is ineligible to claim a deduction for qualified business income if that taxpayer’s taxable income is over the eligible thresholds. At joint taxable income of $827,000, the Shrinks have exceeded the threshold of $464,200.

123
Q
  1. Andrew Albertson has been your client for many years. He is a 69-year-old widower. He has three grown children and his eldest daughter, Andrea, lives in the same town. When you gather data from your client, you learned the names and contact information for his tax and legal advisors. Because he chose to delay claiming his Social Security retirement benefits until he attained age 70, Andrew was paying the monthly premium for his Medicare Part B coverage by personal check. He showed you several late premium notices from the Social Security Administration.
    However, he can’t recall whether or not he actually paid the premium. His coverage could be canceled. In the past year or so, Andrew has been unable to recall many of his activities. You are a CFP ® certificant. Given Andrew’s situation, what would be your best response?

Response:
A. Under the Duty of Confidentiality, do not get involved in Andrew’s possibly omitted payment.

B. Under the Duty of Integrity, you share your concerns with Andrea.

C. Under the Duty of Integrity, discuss your concerns with Andrew’s personal physician.

D. Under the Duty of Diligence, contact the Social Security Administration to learn the status of Andrew’s Medicare Part B premiums and coverage.

A

C. Under the Duty of Integrity, discuss your concerns with Andrew’s personal physician.

Feedback:
At times the CFP® practitioner must weigh the Duty of Integrity against the Duty of Confidentiality. Integrity requires putting the client’s interest first and it is important that Andrew not lose his Medicare Part B Coverage. Discussing Andrew’s late premium payment with his daughter would blatantly violate the Duty of Confidentiality. However, discussing your concerns with Andrew’s doctor is a reasonable choice because the doctor is required to maintain confidentiality under HIPAA and to serve the interest of patients under the Hippocratic Oath.

124
Q
  1. Pete Austin owns Pane in the Glass which he operates as a sole proprietor. Pane in the Glass provides replacement glass and board up services. In the current year, Pane in the Glass generated $120,000 in net Schedule C profits all of which is qualified business income (QBI). With $15,000 of interest income in addition to his earned income, Pete’s current year AGI is $135,000. Which statement below is correct?

Response:
A. While limited and general partnerships, as well as LLCs are treated as pass-through entities, sole proprietorships are not.
B. Under Section 199 A, Pete may claim an above-the-line deduction of $24,000.
C. Under Section 199 A, Pete may claim a below-the-line-deduction of $27,000.
D. Under Section 199 A, Pete may claim a below-the-line deduction of $24,000 even if he claims the standard deduction.

A

D. Under Section 199 A, Pete may claim a below-the-line deduction of $24,000 even if he claims the standard deduction.

Feedback:
Under Section 199-A, Pete may claim a below-the-line deduction of $24,000 even if he claims the standard deduction. A below-the-line deduction without requiring itemizing is an unusual tax treatment that is likely to be favorable to the taxpayer. The $24,000 represents 20 percent of his qualified business income generated through his pass-through business. A sole proprietorship is classified as a pass-through entity for purposes of claiming the 20 percent deduction.

125
Q
  1. Dasch Hund, a married taxpayer, owns and operates Spaniel Spas, an exclusive dog grooming spa. Spaniel Spas operates as an S corporation. Spaniel Spas has one full time employee, Clem Clipper, who will earn a current year salary of $60,000. Spaniel Spas will generate $350,000 in qualified income this year. Dasch’s wife, Fifi, is a novelist. In the current year, she received substantial royalties from sales of Fifty Shades of Financial Planning. The Hund’s current year taxable income is $775,000. Their AGI is $800,000. Regarding this situation, which statement below is correct?

Response:
A. Dasch can claim a QBI deduction of $30,000.
B. Dasch can claim a QBI deduction of $70,000.
C. Dasch can claim a QBI deduction of $95,000.
D. Due to the amount of their AGI no QBI deduction is available.

A

A. Dasch can claim a QBI deduction of $30,000.

Because the Hund’s taxable income exceeds the phaseout threshold of $464,200, their deduction is now limited to the lesser of 20 percent of business income or 50 percent of W-2 wages. Thus, the QBI deduction is only $30,000.

126
Q

Question:
83. Which of the following taxpayers would be eligible for the Retirement Savings Contributions credit (Saver’s credit)?

Response:
A. Married couple (AGI 125,000) saving $6,000 into a Roth IRA
B. Single person (AGI 47,000) deferring $4,000 into a 401K plan at work
C. Single person (AGI 31,000) adding $3,000 to a 529 plan
D. Married couple (AGI 63,000) deferring $7,000 into a 401K plan to get the employer match

A

D. Married couple (AGI 63,000) deferring $7,000 into a 401K plan to get the employer match

Feedback:
The Retirement Savings Contributions credit is designed for low to moderate income taxpayers and matches a percentage (based on AGI) of the taxpayers IRA, retirement plan or
ABLE account contributions but not 529s. The maximum credit is $2,000 (single) and $4,000 (MFJ) and no credit is available for taxpayers over $33,000 (single) and $66,000 (MFJ) AGI.

127
Q
  1. A CFP® professional is discussing an adjustable mortgage with a client. What rate is used most often as the base rate for resetting the mortgage rate?

Response:
A. GNMA
B. SOFR
C. Libor
D. Prime rate

A

B. SOFR
Feedback:
The Secured Overnight Financing Rate (SOFR) is the base rate used for many loans and financial derivatives (LIBOR was used in the past). One possible example could be a mortgage readjusting using the SOR on a specific date plus 300 basis points.

128
Q

Question:
1. An employer can self-fund certain benefits under a 501(c)(9) voluntary employees’ beneficiary association (VEBA). Which of the following may be funded?

I. Death benefits
Il. Medical benefits
III. Unemployment benefits
IV. Retirement benefits
V. Deferred compensation benefits

Response:
A. I, II, III, IV
B. I, II, III
C. I, II
D. IV, V
E. All of the above

A

B. I, II, III

Feedback:
Retirement and deferred compensation benefits may not be funded through the vehicle.

129
Q
  1. You are a CFP® professional. Terry Hand is an old college friend and has been a client of yours for years. He has titled all of his accounts individually in his own name, you haven’t socialized with him since the college years, and you have never met his wife. On a late Friday afternoon, he comes by to ask if you could join him for a drink. You are surprised because, for decades, your relationship has always been as a financial adviser. At a quiet bar he starts belting down drinks. You nurse one. Finally, he says he has a problem. He has two families with children by both his wife and girlfriend. The girlfriend knows he is married. He is worried as to how to set up his estate plan. How would you best respond?

Response:
A. He should place all his assets in a revocable trust to keep all transactions private.
B. He should see an attorney
C. He needs to find another financial adviser because you have a conflict of interest.
D. You need to get his wife’s consent to do an estate plan.

A

B. He should see an attorney

Feedback:
You do not have a conflict of interest or an ethical obligation to involve his wife because she has never been your client. The dynamics of his personal life are complicated. An attorney can advise him on complicated strategies and help him execute documents as needed. You cannot help Terry with documents. There is no need to terminate the relationship. His wife is not your client and his assets appear to be individually owned

130
Q

Baker, Inc. provides a qualified retirement plan (employer funded). The plan falls under numerous ERISA rules. The plan lost 50% due to poor investment decisions in the previous year. What recourse can the employees take?
Response:

A. Sue the plan officials for 100% of the investment losses
B. Sue the plan officials for 50% of the 50% loss
C. Do nothing: qualified plan investment managers are not required to make profits
D. Sue the plan officials for 100% of the losses plus punitive damages
E. Sue the plan officials for losses to the plan

A

E. Sue the plan officials for losses to the plan

Feedback:
Errant plan officials can be held personally liable for losses to the plan as well as other factors. ERISA prohibits monetary punitive damages for claims.

131
Q
  1. Mrs. Elbert, your current client, has just turned 68 years old. She has $10,000 in the bank, modest yearly income of $20,000 from a plan sponsored by her former employer and her Social Security Retirement benefits. In a typical year, Mrs. Elbert spends $10,000 in medical and limited mobility-related expenses. She is generally opposed to living in a nursing home and wants advice on long-term care insurance policies. As a CFP® practitioner, what should you do?

Response:
A. Convince her to purchase a LTC policy
B. Investigate long-term care policies for her, explain LTC and other alternatives to her, enabling her to make a decision
C. Explain Medicare and Medicaid benefits to which she may be entitled.
D. Tell her that it seems as if she cannot afford long-term care insurance and terminate your professional relationship with her.

A

B. Investigate long-term care policies for her, explain LTC and other alternatives to her, enabling her to make a decision

Feedback:
The question says you are a CFP® practitioner. Your client asked for advice on long-term care insurance policies. You should give her all the alternatives. CFP Board wants us to educate clients.

132
Q

Question:
17. Mr. Sims purchased a $500,000 life policy in 1987 paying a single premium of $50,000. The contract cash value has grown to $110,000. He has decided to surrender the contract this year.

Which of the following is true?
Response:

A. $50,000 of the $110,000 will be income tax free; the remaining $60,000 will be subject to tax at ordinary income tax rates.
B. $50,000 of the $110,000 will be income tax free; the remaining $60,000 will be subject to tax at capital gains rate.
C. $60,000 will be subject to tax at ordinary income tax rates plus a 10% penalty.
D. $110,000 will be subject to tax at ordinary income tax rates.

A

Response:
A. $50,000 of the $110,000 will be income tax free; the remaining $60,000 will be subject to tax at ordinary income tax rates.

Feedback:
The policy is not a MEC; therefore, the cash value in excess of basis ($50,000) will be subject to tax at ordinary income tax rates, but not the 10% penalty. 2023-36 years in 1987. The policy was acquired before MEC rules took effect in 1988.

133
Q
  1. Bob works for Technotalk, Inc. Bob’s salary is $100,000. He makes an elective deferral of $22,500 to the company’s 401(k) plan. If Technotalk is a large company, what is the maximum it could contribute and deduct as a match and a profit-sharing contribution for Bob in 2023?

Response:
A. $14,000
B. $22,500
C. $30,000
D. $43,500
E. $66,000

A

D. $43,500

Feedback:
Section 415 of the Internal Revenue Code limits the annual addition to a maximum of 100% of compensation or $66,000. $66,000-$22,500=$43,500. The company may contribute and deduct more than 25% of salary in addition to the elective deferral of an individual employee/participant (not to exceed $66,000) providing that the plan deduction for total includible compensation does not exceed 25% (elective deferrals).

134
Q

Question:
42. Bill Williams, CFP®, wrote a plan for his client, Sally Linton, age 58. Sally indicated she would work for 9 more until her FRA (full retirement age) to qualify for full Social Security and maximum qualified plan benefits. One month later Sally unexpectedly quit her job. The facts are, however, the company she worked for was sold to a competitor and her position was eliminated. The new company offered her an unacceptable position. Now Sally realizes, at 58, with outdated skills, retirement is her only option. Sally indicates to Bill she is willing to sell her second home at the beach. How should Bill proceed?

Response:
A. Make no recommendation until Sally sells the second home.
B. Advise Sally to go back to her employer and take the position.
C. Review Sally’s current lifestyle and expenses and establish a budget until the second home sells.
D. Advise Sally to apply for unemployment benefits.
E. Ask Sally to come back in a few days. The planning needs to be reevaluated.

A

Question:

C. Review Sally’s current lifestyle and expenses and establish a budget until the second home sells.
Feedback:
Bill has all of Sally’s data. He just completed a plan. Sally is upset. She needs advice now. Making her wait might make her more upset. In creating the plan, Bill did assess Sally’s current lifestyle and expenses. Sally can’t file for unemployment insurance benefits because she terminated voluntarily. Sally clearly does not want to take the newly offered position.

135
Q
  1. Sam Waters, age 63, has decided to retire. The company he has worked for maintains an endorsement type split dollar life insurance arrangement on his life. Sam has no other life insurance and has been told that due to several health problems if he applied for life insurance he would be rated (increased premium) or declined coverage altogether. His employer has informed him that he can purchase the policy that is currently held under the split dollar agreement. Presuming that Sam does buy the policy from his employer, what amount would he have to pay for it?

Universal Life Current death benefit - $1,000,000
Cash Value - $120,000
Premiums paid - $100,000

Response:
A. He can purchase the policy by paying $100,000 but it would trigger transfer-for-value income tax consequences.
B. $100,000
C. $120,000
D. $100,000 plus 6% interest. The 6% reflects the employer’s interest charge
E. $120,000 plus 6% interest. The 6% reflects the employer’s interest charge.

A

C. $120,000

Feedback:
Under this endorsement method split dollar life insurance arrangement, if Sam wants to buy the policy under which he is the named insured, he will have to pay the higher of the cash value or premiums paid. The company will realize a gain representing the excess of the cash value over the premiums paid. The endorsement split dollar agreement typically precludes the employer charging interest to the insured employee.

136
Q

Score 0 of 1
Question:
46. Your married client, Mr. Hart, has a son, Robert. Robert is about to enroll in college to study pharmacology. Mr. and Mrs. Hart are delighted that Robert has turned his life around. Only two years ago, Robert was convicted of a felony for distributing a controlled substance. Which among the following education financing strategies is available to the Harts if their AGl is around $100,000?
Response:

A. Claim the American Opportunity Credit
B. Make a deductible charitable gift of tuition to the college
C. Claim the Lifetime Learning Credit
D. Make a tax-free gift by paying Robert’s tuition

A

C. Claim the Lifetime Learning Credit
Feedback:

Both the American Opportunity credit and Lifetime Credit programs specify certain exclusions. For the American Opportunity Credit only, an otherwise eligible student can be excluded if convicted of a felony (in this case distributing a controlled substance). This restriction does not apply to the Lifetime Learning Credit. The Hart’s relatively low joint AGI qualifies them for a Lifetime Learning Credit. Answer D is unclear because it does not indicate whether the tuition is to be paid directly to the education provider. If it is paid to Robert, all or part of the gift may be exposed to federal gift tax. Regarding Answer D. No charitable gift deduction is available for tuition payments to a school.

137
Q
  1. Under an IRC Section 6166 election, which is the following is true?

I. It may only be elected if the client dies owning a business.
Il. It cannot be elected if the business owned by the taxpayer was operated as a sole proprietorship.
III. During the first 4 years the estate is only required to pay interest on any estate tax owed due to the death of the client.
IV. The gross estate must include an interest classified as “closely held” business interests), the value of which exceeds 50% of the gross estate.
V. A section 6166 election allows for installment payment of estate taxes over 10 equal installments beginning 4 years after the clients death.

Response:
A. 1, II, III, V
B. I, III, V
C. III, IV
D. II, IV

A

B. I, III, V

Feedback:
A section 6166 election is available for deceased taxpayers who owned one or more sole-proprietorship business interests. The aggregate holdings in closely held business must represent 35% of the adjusted gross estate. The 2% interest rate only applies to a limited dollar amount of estate tax owed. While the remainder of the estate tax due also qualifies for interest only payments, the rate of interest may be higher than 2%

138
Q
  1. Robert Zimmerman owns Smokey Bacon, Inc. Smokey Bacon provides a profit sharing 401(k). Robert makes the maximum elective deferral and with the company match and typical forfeitures, annual additions have ranged between $20,000 - $25,000.

He has started another company, Eggcellent Eggs, Inc. with some good friends, and they are considering a profit sharing 401(k) plan for Eggcellent Eggs. Robert will be a controlling shareholder in Eggcellent Eggs. Given Robert’s positions, which of the following statements is true?

Response:
A. Robert cannot be a participant in Eggcellent Eggs profit sharing 401(k) plan.
B. Since Robert is fully participating in the Smokey Bacon profit sharong plan, he cannot participate in Eggcellent Eggs profit sharing plan (related employers).
C. Robert is limited to the lesser of 25% of covered compensation or $61,000 (2022) for annual additions provided by both Bacon and Eggs.
D. He cannot defer any compensation into Eggcellent Eggs 401(k) plan.

A

D. He cannot defer any compensation into Eggcellent Eggs 401(k) plan.

Feedback:
Robert can be a participant in Eggcellent Eggs plan for profit sharing contributions but not elect any more deferrals. Answer C is incorrect because his annual additions limit (for both plans combined) is the lesser of 100% of compensation or $66,000 in 2023 (Bacon and Eggs are related employers).

139
Q
  1. Mr. Able, age 75, came to you, a CFP® professional, with large folders filled with investment data. He is confused. He has concerns regarding all the investments his advisor has sold him. He would like you to help him identify which investments have been performing better over the last 10+ years. You tell him that because this analysis will take a considerable amount of time, you would need to charge him a fee. Mr. Able agrees. As you proceed you realize that the investments are specific common stocks or bonds. The allocation reflects the traditional 70/30 split. Over the prior decade, on average, the stock portion generated returns of over 10% and the bond portion generated returns of 4%. How should you best respond to Mr. Able’s concerns?

Response:
A. Given his age, advise Mr. Able to rebalance the investments for a 60/40 split allocation.
B. The bond return indicates excessive risk for a 75 year old client.
C. You should discuss Mr. Able’s health because the return levels may have to continue for the next decade.
Feedback:
D. Mr. Able should keep his advisor.

A

C. You should discuss Mr. Able’s health because the return levels may have to continue for the next decade.

Feedback:
Mr. Able’s health and life expectancy should factor into his investment plans. Given strong health and a family history of longevity, he might live for another 20 years. In light of historical returns in the decade from 2009 to 2022, the overall portfolio return is certainly acceptable. The advisor has not underperformed.

140
Q
  1. The privilege of gift splitting only applies when:

Response:
A. A gift is made by a man and woman living together.
B. A gift is made by a woman (single) before she is married later in that same calendar year.
C. A gift is made after the spouses are legally divorced.
D. A gift is made before one spouse dies later in that calendar year.

A

D. A gift is made before one spouse dies later in that calendar year.

Feedback:
Only married couples may split gifts. If one spouse dies in the year of the gift, the split is generally allowed if the gift was made before the decedent actually died.

141
Q

Question:
62. Bill works for two related employers. Each employer provides a 401(k) plan. With his compensation from his night job is $50,000 and his compensation from his day job is $60,000. If the plan for each company allows a 6% deferral and provides a 3% match, how much can Bill defer in the current tax year?

Response:
A. $3,000
B. $3,300
C. $6,600
D. $13,200
E. $22,500

A

C. $6,600

Feedback:
6% of $110,000 An employer plan document can put a limit on the Elective deferral. The plan limits elective deferrals to 6% of Bill’s compensation. Thus, $22,500 is not a supportable answer because of the plan limitation of 6%

142
Q
  1. George Able, age 60, enjoys working and has no wish to retire. He is researching businesses he could buy. Given that entrepreneuring is risky, George’s required rate of return is 7% to 8%. If a business earned an annual net profit of $100,000, what should George pay for the business?

Response:
A. $1,000,000
B. $1,250,000
C. $1,333,333
D. $1,428,571

A

B. $1,250,000

Feedback:
Using the higher rate 8% produces a lower number ($1,250,000). George would prefer to pay less to acquire the risky business. Using the 7% produces a maximum number.
($1,428,571) $100,000 = $1,250,000.08

143
Q
  1. Dolores Delmar established a special needs trust for her developmentally disabled child. Which of the following is true?

Response:
A. Dolores should serve as the trustee of the trust.
B. The trustee can reposition the assets in the trust.
C. A family member may serve as be trustee of the trust.
D. Investment income cannot be used to buy life insurance on Dolores.

A

B. The trustee can reposition the assets in the trust.

Feedback:
In general, trustees may reposition trust assets. If Dolores serves trustee, that she is controlling the beneficial enjoyment of the trust’s assets will taint the trust for income and estate taxes. A family member can be a co-trustee with an independent trustee. Life insurance can be purchased. However, using income from the trust to pay the premiums will taint the trust for income tax purposes. Life insurance on the grantor of a special needs trust that is held within that trust is a typical occurrence.

144
Q

Score 1 of 1
Question:
74. Joan Lundy and Judy Baker own a successful business together. They want to create a buy-sell arrangement funded by life insurance. They are both in their early fifties and in good health.

They want the arrangement to cover death, retirement, or one of them leaving the business. Which of the arrangements and policies make the most sense?

Response:
A. An entity purchase buy-sell agreement funded with whole life insurance policies
B. An entity purchase buy-sell agreement funded with universal life policies featuring Option A
C. A cross-purchase buy-sell agreement funded with whole life insurance policies
D. A cross-purchase buy-sell agreement funded with universal life policies featuring Option A
E. A handshake agreement that is funded with 20-year level term insurance.

A

C. A cross-purchase buy-sell agreement funded with whole life insurance policies

Feedback:
The only way the surviving owner will receive a step-up in basis is to implement the cross-purchase buy-sell arrangement. Whole life insurance has a level premium which will build the cash value for one owner to buy the stock of the (other) retiring or departing owner. Joan and Judy are both in good health. They indicate no immediate plans to retire or leave the business, so the policy may have to last until they reach age 80 or 90.

145
Q
  1. Mark Thomas hired you to be his financial planner. Mark maintains 2503(c) trust for his 2-year-old daughter, June. Mark named himself as trustee because he wants discretion over the distribution income and principal. The trust instrument includes the provision that any undistributed income and principal will be distributed to June when she turns age 21. After reviewing the trust agreement what is the most serious concern that you would share with Mark?
    Response:

A. Mark should consult with an attorney and have the trust amended to remove himself as trustee.
B. The trust needs to invest in growth securities because trust income tax rates hit 37% at relatively low levels of income.
C. That he should confirm whether he really wants his daughter to get the trust principal when she reaches age 21.
D. That the trust is actually a 2503(b) trust and not a 2503(c) trust

A

A. Mark should consult with an attorney and have the trust amended to remove himself as trustee.

Feedback:
If Mark continues as trustee of the trust into which he has transferred his own the 2503(c) is a tainted trust for both estate and income tax purposes. The notion of minimizing income and subsequent tax is important, but the trustee situation is more worrisome. The trust document needs to be amended or voided. This is a 2503(c) trust.

146
Q
  1. Joan purchased a single premium whole life insurance policy in 2005. She paid one $30,000 premium for the coverage. The policy’s death benefit is $100,000. Today, the contract is worth $50,000 represented by $40,000 guaranteed cash value and $10,000 of dividend cash value. If Joan takes out a policy loan of $30,000, which of the following is true?

Response:
A. She can receive the $10,000 of dividends tax-free, and $20,000 will be subject to ordinary income tax plus a 10% penalty.
She can borrow $20,000 tax-free, but B. $10,000 will be subject to ordinary income taxes plus a 10% penalty.
C. $20,000 will be subject to ordinary income tax plus a 10% penalty, and $10,000 will be a tax-free return of basis.
D. The entire $30,000 will be subject to ordinary income tax plus a 10% penalty.

A

C. $20,000 will be subject to ordinary income tax plus a 10% penalty, and $10,000 will be a tax-free return of basis.

Feedback:
A single premium policy purchased after 1988 is a MEC. Current CV $50,000 - basis $30,000 = $20,000 gain.
Dividends in a MEC become taxable when borrowed or withdrawn. Joan’s gain is $20,000; $10,000 is her true basis in the policy. Dividends under a MEC are generally taxed.
Only the gain is subject to the 10% penalty in addition to tax at ordinary rates.

147
Q
  1. Lamar and Abby Sanford, who have three teenage and pre-teen children, are confused about the difference between the American Opportunity Credit (AOC) and the Lifetime Learning Credit. How would you answer the Sanfords?

Response:
A. If you elect a full Lifetime Learning Credit, you cannot claim an AOC for the same expense in the same year.
B. The maximum amount of AOC is $2,000 plus 25% of the next $2,000 for a total of $2,500 per tax year. The maximum amount of the Lifetime Learning Credit is $2,000 per year.
C. The Lifetime Learning Credit is no longer available after the student turns age 30..
D. The AOC is available for both undergraduate and graduate post-secondary education.

A

Response:
A. If you elect a full Lifetime Learning Credit, you cannot claim an AOC for the same expense in the same year.

Feedback:
Answer B is incorrect. The AOC is per eligible student per year. If a family has three children in college in the same year, each is eligible for the full AOC. Lifetime is maximum per year.
The AOC is available only for the first four years of post-secondary education. No age limitation applies to the Lifetime Learning Credit.

148
Q

Can you still file a 1040 EZ?

A

No, the TCJA eliminated the 1040 EZ

149
Q
  1. Which of the following are amounts received by the owner of a life insurance policy that would be treated as income-first distributions under a MEC contract?

I. Cash dividends
II. Interest accrued on a policy loan (added to the loan balance)
III. Dividends retained by the insurer to purchase “paid-up” additions
IV. Dividends retained by the insurer as principal or interest to pay off a policy loan

Response:
A. All of the above
B. I, II, V
C. II, III
D. III, IV

A

B. I, II, V

Feedback:
When policy dividends from modified endowment contracts (MECs) are used like cash, they are distributed under FIFO rules such as those applicable to annuity distributions. Cash distributions and dividends used to pay off policy loans are considered to be income first distributions. The interest accruing from a policy loan on a MEC is treated the same way for federal tax purposes. Dividends used to buy paid up (permanent) additions are not treated as income-first distributions. The loan interest was not distributed. It was added to the outstandting loan balance.

150
Q

What do revocable trusts and a 2503C minors trust ultimately pass into?

A

A family trust - irrevocable

151
Q
  1. The Wonder Widget Company operates as a C Corporation. Its officers are concerned about their personal liability exposure on becoming appointed as trustees for the company’s pension plan. How should their concerns be minimized?

Response:
A. Provide four different families of funds for participant investment choices
B. Provide eight different mutual funds for participant investment choices
C. Offer mutual funds that range in risk level and composition from money market accounts to sector funds
D. Interview then hire an investment manager to manage their plan’s portfolio

A

D. Interview then hire an investment manager to manage their plan’s portfolio

Feedback:
If a suitable investment manager has been appointed, generally the trustee will not be liable for the acts or omissions of that investment manager. However, this relief does not totally excuse from liability the named fiduciary who appointed the investment manager.

152
Q
  1. A FICA covered worker will be entitled to Social Security disability benefits if which of the following conditions apply?

I. The worker has been disabled for 12 months or is expected to be disabled for at least 12 months or has a disability that is expected to result in death
II. The worker has attained FRA or is older
III. The worker is insured for disability benefits
IV. The worker has filed an application for disability benefits
V. The worker has completed a 5-month waiting period or is exempted from this requirement

Response:
A. All of the above
B. I, III, IV, V
C. II, III, IV
D. III, V
E. IV, V

A

B. I, III, IV, V
Feedback:
To be eligible to claim Social Security disability income benefits, the FICA covered worker must be younger than full retirement age (FRA). At and after FRA, only retirement benefits are available.

153
Q
  1. At age 55, Harry started taking substantially equal payments from his IRA. For three consecutive years, he took the required amount. Then, in year four, due to a qualified hardship (to prevent foreclosure on his home), he withdrew $50,000 more than the normal substantial equal payment under the annuity method of distribution he had selected. What, if any, amount of penalty did Harry have to pay on the $50,000?

Response:
A. None, because the $50,000 excess withdrawal was for a bona fide financial hardship.
B. None, because Harry made a qualified, self-directed loan of $50,000
C. 10% of the $50,000 excess distribution
D. 10% of the distributions received in years 1, 2, and 3, and the $50,000 excess distribution in year 4
E. There was no penalty because he was 59 ½

A

C. 10% of the $50,000 excess distribution

Feedback:
The question specifically addresses the $50,000 excess withdrawals rather than the compliant withdrawals from years 1-3 (which will also incur penalty). Hardship withdrawals are never available from IRAs as they are from 401(k) plans and certain 403(b) arrangements. The question says Harry is 59, not 59 ½.

154
Q
  1. Your client, Alan Stephens, is a loving father, age 45. He is about to co-sign a mortgage ($300,000) for a home his daughter, Tiffany, plans to acquire. You are aware the daughter has a history of financial problems since graduating from college. As Alan’s planner, you should:

Response:
A. Ask for a meeting with both Alan and Tiffany
B. Advise him not to co-sign the loan with Tiffany
C. Be silent regarding your concerns. it is not appropriate for you to meddle in family business.
D. Recommend family counseling for Tiffany

A

A. Ask for a meeting with both Alan and Tiffany

Feedback:
Some dialogue about finances between Alan and his daughter would make sense and you can facilitate the discussion. Answer B makes better sense after the joint meeting. Is being silent about your concerns serving the needs of your client, Alan? Your concern is Alan. Tiffany is not your client.

155
Q
  1. George and Linda gifted $110,000 to an irrevocable living trust that includes Crummey provisions. The trust has named their two twin nephews, Larry and Barry as its beneficiaries. George and Linda consent to gift splitting. When George or Linda dies, how much, if any, of the gift will be brought back into their gross estate(s)?

Response:
A. $0
B. $21,000
C. $42,000
D. $110,000

A

Response:
A. $0

Feedback:
A gift of cash to an irrevocable living trust is irrevocable. The gift is removed from their gross estates for federal estate tax purposes. While both George and Linda will have made a taxable gift of $21,000, that is not reflected in their gross estate(s). Rather, it is added to their taxable estate(s) to arrive at the tax base.
($110,000 - [68,000 (17,000 x 4)] = $42,000 42,000 / 2 = 21,000 each

156
Q
  1. Former college roommates, Barbara and Kathy, are 50/50 owners of BK, Inc. When they started BK, the corporation acquired two $1,000,000 face value key-person term life insurance policies. Barbara is the named insured in one policy and Kathy in the other. Over the years, the company has paid $5,000 in premium on Barbara’s policy and $4,000 in premium on Kathy’s policy. Barbara and Kathy have decided to use the policies to back a new cross purchase buy-sell agreement. If Barbara buys the policy in which Kathy is the named insured paid by BK, what will be the tax outcome if Kathy dies within one year following Barbara’s acquisition of the policy?

Response:
A. Barbara will receive $1,000,000 income tax-free (term insurance exclusion).
B. The policy will be subject to the transfer for value rules making the death benefit net of basis subject to federal income tax.
C. The policy will be included in Kathy’s gross estate for federal tax purposes (3-year rule).
D. The policy’s death proceeds will increase the fair market value of BK, Inc. by $1,000,000.

A

B. The policy will be subject to the transfer for value rules making the death benefit net of basis subject to federal income tax.

Feedback:
The policy was sold to someone other than the insured or to a business in which the buyer is an owner or partner. Whether the policy is term or permanent insurance is immaterial to the rule. Transfer for value rules makes the policy’s death benefits income taxable to the beneficiary to the extent that it exceeds basis. The 3-year rule doesn’t apply when a sale (transfer for value) occurs.

157
Q

Dr. Perkins is about to retire. The Brain Trust Practice Group is willing to transfer the key person policy to him for its current fair market value. Dr. Perkins has concerns about the gift income and estate tax ramifications relative to these life insurance policies under which he is now insured. Which of the following strategies is (are) the best option(s) for Dr. Perkins if he lives for at least three more years?

I. Establish an irrevocable life insurance trust to purchase the employer-paid policy.
Il. Have his wife purchase the employer-paid policy.
III. Gift the personally owned life insurance policy to his wife
IV. Gift the personally owned policy to his life insurance trust (His wife and two daughters its beneficiaries.)
V. Buy the employer-paid key person life insurance policy and then gift the policy to his life insurance trust

A. All of the above
B. II, III
C. II, IV, V
D. III, IV
E. IV, V

A

E. IV, V

Feedback:
Answers I and Il will trigger “transfer for value” rules and make the policy’s death benefit subject to income tax. If Dr. Perkins gifts the policy to his wife, it will be includible it in her gross estate for federal estate tax purposes thus increasing potential estate tax at the second death. Answer E is the best answer to get the policies out of his wife’s estate. The exact amount of taxable gift less exclusion is not part of the answer. Since his wife is a beneficiary, it is unlikely that gift splitting is permissible. But, his estate tax applicable amount remains available.

158
Q

Question:
Score 0 of 1
45. Mrs. Tilden, a widow, has gifted extensively to her daughter, Sally. She used her entire gift property exemption amount and actually paid federal gift tax on her most recent gifts. Mrs. Tilden recently married Bill Widner. She is considering gifting him $1,000,000 with the written understanding that he will then gift the $1,000,000 to Sally. How would you respond after she explains her strategy?

Response:
A. This is an effective way to accomplish effective gift tax planning.
B. The transfer of $1,000,000 reduced by one annual gift tax exclusion from Mrs. Tilden to Bill Widner will be a taxable gift.
C. Mrs. Tilden and Bill Widner need to be married for one year for this technique to work.
D. The trasnfer of $1,000,000 reduced by one annual gift tax exclusion from Mrs. Tilden to Bill Widner will be a non-taxable gift.

A

B. The transfer of $1,000,000 reduced by one annual gift tax exclusion from Mrs. Tilden to Bill Widner will be a taxable gift.

Feedback:

It appears that the gift will not qualify for the marital deduction. To qualify for the deduction, the done spouse must be given the property outright or must have at least a right to the income from the property and a general power of appointment over the principal. The IRS would consider this to be a step transaction and thus a fraudulent transfer.

159
Q
  1. Your client, Mr. Smith, purchased and sold the following stocks during a two-year period. Which situation or situations below created a “wash” sale?
    NOTE: All transactions are 100 share round lots.
  2. March 1st purchased ABC @ $15; December 1st purchased ABC @ $10; December 31st sold ABC @ $10
    II. November 30th purchased LMN @ $50; December 15th purchased LMN @ $52; December 29th sold LMN @ $54
    III. January 1st purchased XYZ @ $60; February 15th sold XYZ @ $50; March 16th purchased XYZ at $52

Response:
A. I, II, I
B. I, III
C. I, II
D. Il
E. III

A

B. I, III

Feedback:
The ABC and the XYZ transactions violate the wash sale rules.
–ABC’s basis is $10 cost plus $5 recognized loss. (December 1st and December 31st)
–XYZ’s basis is $52 cost plus $10 recognized loss. (February 15 and March 16). There are only 28/29 days in Feburary
– LMN was sold for a gain.

160
Q
  1. Mrs. Teal has come to you, a CFP® practitioner, for advice. She is age 75 and in excellent health. Mrs. Teal’s mother lived to 94. She is unsure about her father’s life expectancy because he died in the war. Mrs. Teal is concerned because her husband died about 3 months ago. He handled all of their financial affairs. Now widowed, she has tried over the past 3 months to figure out her financial future, but the information on her account statements is overwhelming. Which of the following items should Mrs. Teal worry about least?

Response:
A. Being able to continue to live in her current home.
B. Whether cash flow will cover her expenses
C. Whether she can keep the vacation home on the lake for summer outings to spend with her children and grandchildren.
D. Her investment risk tolerance

A

C. Whether she can keep the vacation home on the lake for summer outings to spend with her children and grandchildren.

Feedback:
Although Mrs. Teal will be disappointed if she can’t maintain the summer vacation home, the other issues are more pressing. Her cash flow (income) must consider whether she can continue to live in her current home. Her risk tolerance will affect the amount of income her investments would generate.

161
Q

Question:
57. Your client, Sam Glover, purchased a property for $2,000,000. Sam did not maintain the property particularly well. He claimed depreciation deductions and the adjusted basis is now $1,117,000. Sam has now decided to gift the property to his son. Sam has already used his full gift tax exemption. He had to pay $440,000 in gift tax (40% bracket). His son, who is very handy and a hard worker, just sold the property for $1,500,000. What amount of gain will his son have to report from his sale of the property?

Response:

A. $85,000
B. $383,000
C. $440,000
D. $885,000
Е. -0-

A

B. $383,000

Feedback:
Because this is not loss property, the son’s basis will be Sam’s adjusted basis ($1,117,000). (The basis is lower than original due to the CRDs not the market value of the property) The son’s gain is $383,000. The annual gift exclusion never affects basis. The gift taxes paid would only adjust basis if the property had appreciated. This is depreciated property. The gift taxes paid only increase the basis when the FMV of the property as of the date of the gift exceeds the donor’s original cost basis.

162
Q
  1. Corporate annual reports would generally not include which of the following?

Response:
A. Depreciation methods
B. Stock options
C. Profitability projections
D. Inventory methods
E. Outlook for the firm’s products in various industries in which it operates

A

C. Profitability projections

Feedback:
Profitability projections should not be included in corporate annual reports. The SEC believes that such projections could mislead shareholders and others.

163
Q
  1. Mr. and Mrs. Grandparent have paid $50,000 into their granddaughter’s “prepaid tuition program.” The arrangement qualifies as a Section 529 program. Which of the following is true?
    Response:

A. Prepaid tuition plans may only pay for tuition and mandatory fees.
B. Prepaid tuition plans do not affect the expected family contribution for determining available financial aid.
C. Prepaid tuition plans do not allow the beneficiary to attend a private or out-of-state college.
D. Prepaid tuition plans provide a rate of return that is linked directly to the return on the securities in which the prepaid tuition plan invests.
E. If the qualified higher education expenses (QHEE) are less than the total distributions, all the distributions from the prepaid plan are then treated as taxable income.

A

A. Prepaid tuition plans may only pay for tuition and mandatory fees.

Feedback:
If the beneficiary of a prepaid tuition (529) plan attends a private or out-of-state college, the program will determine the value of the contract. If the QHEEs are equal to, or greater than, the total distribution, they are tax-free. Prepaid tuition plans pay for tuition and mandatory fees. They do not cover room and board. Prepaid tuition plans do affect the expected family contributions.

164
Q
  1. Which of the following statements is true about a medical expense flexible spending account (FSA)?

Response:
A. If there is money left in the care account at the end of the year, it can be used for dependent care.
B. An FSA can discriminate in favor of HCEs if less than 70% of the employees participate in the plan.
C. The employee must elect in writing the amount of the salary reduction in the calendar year prior to the year in which that reduction is to take effect.
D. Funds that are forfeited when an employee fails to withdraw them during the allotted time revert directly to the employer for general operations.

A

C. The employee must elect in writing the amount of the salary reduction in the calendar year prior to the year in which that reduction is to take effect.

Feedback:
A participant in an FSA plan must elect in writing the amount of the salary reduction in the calendar year prior to the year in which that reduction is to take effect. Unused funds in medical expense FSAs cannot be used for dependent care. An FSA must not discriminate in favor of HCEs. However, there is no 70% participation testing. Forfeited funds in an FSA may not go directly to the employee for general operations but they can be used to defray future administrative expenses of the FSA itself.

165
Q
  1. After finding that a CFP® certificant violated the CFP® Code of Ethics, the CFP® Board’s Disciplinary and Ethics Commission can impose which form of discipline?

I. Private censure
II. Public letter of admonition
III. Suspension
IV. Revocation

Response:
A. All the above
B. II,III,IV
C. III, IV
D. None of the above answers

A

A. All the above

Feedback:
Presuming a deemed violation of the CFP® Code of Ethics and Professional Responsibility, the Disciplinary and Ethics Committee may impose all the forms of discipline that are shown.

166
Q
  1. Tilly recently inherited a significant amount of personal property from her mother. She is concerned about her property insurance coverage. Which items) do you suggest that she talk with her insurance agent about full protection?

I. Gold coins
II. Old furniture and workable TVs that are used in what the family calls “The Shack” that is 500 feet away from the main house
III. Silver-flatware service for twelve
IV. Paintings bought on Amazon.com

Response:

A. I, II, III
B. I, III
C. II, III
D. III, IV
E. Il

A

B. I, III
Feedback:

Under the HO forms, coins are covered for no more than $200. While the question does not indicate the coins are collectible, given that they are gold the $200 limit applies. Silver has an internal (sub) limit of $2,500 in coverage for theft only. The personal property in a separate structure (“The Shack”) is covered. Artwork bought on the internet would be covered as normal personal property rather than collectibles.

167
Q
  1. Mrs. Andrews is referred to you by a current client who happens to be her employer. The referring client is a physician and Mrs. Andrews works as the Medicare billing administrator in his office. She is divorced, and her income barely covers her living expenses. She wants to talk with you about her daughter. She asked you to meet with her daughter and son-in-law. You agree. Her daughter, Leslie, and her husband Tommy meet with you. Both are employed in hourly wage jobs and have no savings. Fortunately, their employers provide them with group health insurance.
    Leslie is pregnant and the expected due date is 2 months away. That is all the financial information they can provide. What you recommend that Leslie and Tommy do?

Response:
A. Establish an emergency fund to cover 6 months of fixed and variable expenses.
B. Buy term life insurance on Tommy’s life with a death benefit of at least $500,000.
C. Suggest that Mrs. Andrews tighten her belt so she can help them with money.
D. Help Leslie and Tommy determine whether they qualify for any local or state financial assistance, like food stamps.

A

B. Buy term life insurance on Tommy’s life with a death benefit of at least $500,000.

Feedback:
This growing family will need life insurance. This is especially true because Leslie and Tommy can’t rely on Mrs. Andrews for financial help. With term coverage they should be able to obtain a reasonably high death benefit. With two months before the baby comes and limited wages a six-month emergency fund does not seem reasonable. Further, they will have extra cash needs for baby-related expenses. Answer C is incorrect because Mrs. Andrews is not the client for whom the recommendation in the question is to be made.

168
Q
  1. Which of the following assets are liquid?

Response:
A. A S&P 500 ETF
В. А СМО
C. A GIC
D. A 1-year CD

A

D. A 1-year CD

Feedback:
In the absence of early withdrawal, the CD would not be expected to lose principal. The S&P ETF and CMO are marketable but there could be a substantial change in price. The GIC isn’t liquid.

169
Q
  1. The Federal Reserve Board (FRB) has the means to affect the supply of money in the US economy. If the federal government wanted to increase the money supply, what would it most likely do?

A. Sell Treasury securities
B. Reduce federal income taxes
C. Buy Treasury Securities
D. Reduce M1

A

B. Reduce federal income taxes

Feedback:
represent monetary policy.
The question asks what the federal government would do and not the activities of the FRB. This represents fiscal, rather than monetary policy. The other answers which are incorrect

170
Q

Question:
83. Arthur is looking to buy RST common stock for $100. The current dividend is $2. The stock is expected to grow its dividend by 6% for three years, and then by 8%
thereafter. Arthur’s required rate of return on this stock is 10%. Should he buy the stock? V = D1
r-g
Response:

A. No, based on the DDM, the stock appears to be overvalued.
B. Yes, based on the DDM, the stock appears to be undervalued.
C. Yes, based on the DDM, the stock will satisfy Arthur’s required rate of return.
D. Yes, because the dividends would only be taxed at 15%.

A

B. Yes, based on the DDM, the stock appears to be undervalued.
Feedback:
By factoring the DDM, we know that the current market price of $100 is less than $108.00. V = $2 (1 + .08). = $2.16 = $108.00
.10 -08
.02
In the formula above, the 2nd growth rate is used to calculate the intrinsic value with an adjustment upward or downward relative to whether the early years’ dividend is lower or higher than the later years’ dividend

171
Q
  1. Toby, age 74, has been taking distributions from his qualified plan. Now, to his dismay, he has been sued and lost in court. His only real asset is the qualified plan distributions. Can the plaintiff collect against his qualified plan distributions?

Response:
A. No, ERISA forbids “alienation of plan benefits.”
B. No, Toby can file for Chapter 7 bankruptcy and keep his plan benefits.
C. Yes, but only for a portion of the distributions
D. Yes, because that is his only asset

A

C. Yes, but only for a portion of the distributions

Feedback:
The “anti-alienation” provision only applies during the accumulation period, not after the account goes into pay status. Once the distributions are paid out, they become an asset of the participant, subject to any creditor’s claims that may be pending when the benefits are received.

172
Q

Score 0 of 1
Question:
6. Mark Spout created an irrevocable trust for the benefit of his dependent children. Mark named the local bank as trustee of the trust and authorized it to invest in stocks, bonds, and negotiable certificates of deposit. Included in the investment authority is the right to use trust income to purchase insurance on Mark’s life. All funds are currently invested in high-yielding bonds paying 7% semiannual interest on a par value of $100,000. Twenty-five percent of the bond investment income is being used to pay the premium on a policy on Mark’s life. Which taxpayer must pay tax on the income of the trust and why?

A. The bank because of its broad authority as trustee
B. The children because the income is paid by them
C. The trust because it is irrevocable with no benefits to grantor
D. Mark because of the grantor rules
Feedback:

A

D. Mark because of the grantor rules
Feedback:
If any portion of the trust income is, or may be, used to purchase insurance on the life of the grantor or grantor’s spouse, then the trust is a grantor trust. The trust is/was also used to benefit his dependent children (support). 25% is to purchase insurance and the remainder is used for support.

173
Q
  1. Scott Harding died recently (at age 74), with a taxable estate of roughly $8 million. Some years ago, he had seen an estate tax attorney and completed an extensive estate plan with a variety of trusts. Among his assets is a stone-constructed New England farmhouse having a current fair market value of $2 million. Scott’s will bequeathed the home to Clarisse who, due to authoring a best-selling mystery novel, is worth approximately $20 million in her own right. Nevertheless, when Scott died, Clarisse, (under the guidance of her tax advisor), did not want to own the home outright, fearing it would become part of her already substantial gross estate. However, she wishes to live in the home where she can continue to write and enjoy her grandchildren for the rest of her life. The home had been in the Harding family since its arrival in America in 1789. At the time of his death, it was owned fee simple by Scott. There is great sentiment associated with the home: Scott and his wife, Clarisse (age 66) were married in the home and raised their two children Rebecca and Jonah (now adults) there. What technique, if any, will accomplish Clarisse’s goals?
    Response:
    A. Clarisse cannot have her cake and eat it too: If she disclaims ownership of the home, she may not live there without paying fair market rent.
    B. With a proper provision in Scott’s will, if Clarisse disclaims ownership of the home, it can be transferred to a disclaimer trust, the terms of which permit Clarisse to live in the home for her lifetime. This would be a family trust.
    Feedback:
    Many well written wills include disclaimer trust provisions which give the surviving spouse the ability to put specific disclaimed assets into the trust by disclaiming ownership of a portion of the estate. Disclaimed property interests are transferred to the trust, without being taxed.

Using answer D will mean for the life of the QPRT it could be brought back into her estate. It is not a bad answer. With Answer B, the house uses the exemption. Suggestion: The questions are not too long to read. I would recommend reading it first.
C. With a proper provision in Scott’s will, if Clarisse disclaims ownership of the home, it can be transferred to a disclaimer trust, the terms of which permit Clarisse to live in the home for her lifetime. This would be a marital trust because Clarisse has a life interest in the home.

D. A qualified personal residence trust (QPRT) should accomplish Clarisse’s dual goals of not including Scott’s home in her gross estate and the right to live in the home for her entire lifetime.

A

B. With a proper provision in Scott’s will, if Clarisse disclaims ownership of the home, it can be transferred to a disclaimer trust, the terms of which permit Clarisse to live in the home for her lifetime. This would be a family trust.

Feedback:
Many well written wills include disclaimer trust provisions which give the surviving spouse the ability to put specific disclaimed assets into the trust by disclaiming ownership of a portion of the estate. Disclaimed property interests are transferred to the trust, without being taxed.
Provisions can be written into the trust that provide for regular payouts from the trust to support survivors, or in the case of the Hardings, the right to occupy (but not own) property. The trust can also be written so that surviving minor children can also be provided for, as long as the surviving spouse elects to disclaim inherited assets, passing them on to the trust. To keep the assets from being included in Clarisse’s estate, the trust would have to be a family trust rather than a marital trust. If a disclaimer trust is used, the full extent of the tax planning occurs upon the death of the first spouse. At that point, the surviving spouse can either accept the trust assets or disclaim them. If he or she disclaims them into the disclaimer trust, the trust will function like a credit shelter trust that will “shelter” the assets from inclusion in the surviving spouse’s estate. But if there is no tax reason to use credit shelter planning, the spouse can simply receive the assets outright. This allows tax-planning flexibility without creating unnecessary complication.
Using answer D will mean for the life of the QPRT it could be brought back into her estate. It is not a bad answer. With Answer B, the house uses the exemption.

174
Q

Question:
9. Tom is a participant in an HSA. He wants to pay for LTC premiums with distributions from an HSA. Is he allowed to do that?

Response:
A. No, the HSA can only pay for LTC expenses, not premiums.
B. No, the HSA can only pay for qualified medical expenses.
C. Yes, the HSA can pay for qualified LTC premiums (age-based).
D. Yes, the HSA can pay for all LTC premiums and expenses.

A

C. Yes, the HSA can pay for qualified LTC premiums (age-based).

Feedback:
This is the best answer. The amount of qualified long-term care insurance premiums that constitute qualified expenses is allowed. However, the LTC premiums are still limited to age-based limitations (as adjusted annually). Answer B does not specifically answer the question. This is an HSA not an FSA. Premium payments are not allowed in FSAs but are allowed in HSAs

175
Q
  1. Mrs. Lucy, age 80, is in reasonable health. Five years ago, her husband died leaving her $3 million and placing $3.5 million in a bypass trust for her benefit. In addition, their home was in JTWROS. The home, FMV value $500,000 and the $4 million of investments has a high basis of $3.5 million. Mrs. Lucy has two married children and 5 grandchildren to keep her estate under $5
    million. What type of asset do you recommend she give and to whom?

Response:
A. Low basis, high dividend paying investments to both children and grandchildren.
B. High basis, high dividend paying investments to both children and grandchildren.
C. Low basis, growth investments to children and high basis, growth investments to grandchildren.
D. High basis, growth investments to children and low basis, growth investments to grandchildren.

A

B. High basis, high dividend paying investments to both children and grandchildren.

Feedback:
Think ages. The grandchildren have to be age 30 maybe age 40. With high basis, they can sell the investment with little or no tax or keep it and get big dividends. They could be taxed a 0% or at most 15%. Low basis investments would be subject to capital gains. If she keeps the low basis stocks until death, her estate will get a full step-up in basis. She does not have an estate tax situation.

176
Q
  1. An existing client calls you, a CFP® professional, with a question about a topic you aren’t very familiar with. She tells you her husband served in the military decades ago and is now receiving care in a nursing home. She read a brochure at this new facility about a VA benefit called Aid and Attendance that could pay her around $1,500 per month as the spouse of a serviceman who is receiving skilled nursing care. She would like you to help her qualify for the benefit.
    You tell her that you will do some homework and arrange a meeting with her the following week to discuss their situation further. After doing some research, you discover that there are very stringent income and asset restrictions to qualify for benefits. Since they have been clients for years, you know they have a net worth of around four million dollars. In your meeting you should:

Response:
A. Explain the income and asset thresholds that are well lower than their current situation. Help her understand that this benefit is not intended for people in their financial position and recommend they not pursue the benefit further.
B. Tell your client you aren’t an expert in this matter and put her in touch with an attorney with whom you have worked the past whom you know to be a military veteran.
C. Review options for how the client may effectively reduce income and gift assets to qualify for the benefit.
D. Recommend she purchase long term care insurance and review their current retirement and investment objectives for other options to meet their needs.

A

C. Review options for how the client may effectively reduce income and gift assets to qualify for the benefit.

Feedback:
In D, you never address her objective and discuss options on how to qualify: you only give recommendations apart from it. A is a good answer however you should let her determine if she is willing to go to lengths to qualify, you shouldn’t be making the decision for her by dismissing the option. B would be a great answer but it is unclear if the attorney is competent in the subject. In this meeting, you should review the client’s options as she requested, answer C. Once that is done you may then decide to advise a more suitable solution (Answer D). If you answered B I am not saying you are wrong. This is a practice question. (Subjective)

177
Q
  1. John is a CFP® practitioner who is establishing a relationship with forty-year-old Annie Wong in California. They have met once, and in that meeting John was able to establish and define their relationship and gather all of Annie’s financial data. While in the process of preparing for their second meeting and analyzing the client data he received an email from Annie stating that she had been reassigned to a 6 month project in Japan and will not be able to make their next appointment in which John had planned to present his recommendations to her. Annie would still maintain her California address and residency and US citizenship and intends to return to live in California when the assignment is completed. Since she felt it would be difficult to meet in person or over the phone she has asked John to email her the list of recommended investments for her portfolio. John obliges her request, finishes analyzing her situation and sends her an email containing a list of the investments he recommended for her and the percentages she should place in each investment. A few days later John received another email from Annie in Japan stating that she liked the recommended growth portfolio and had gone online and executed the trades on her own in her account. She thanked John for working with her unique situation and planned to meet with him again sometime after she returned from the overseas assignment. Has John violated any of the CFP® board’s practice standards in this scenario, or is this a suitable approach to her situation?
    Response:
    A. John may have breached the implementation step of the planning process because Annie executing the trades online is not a suitable form of implementation.
    B. John may have breached the monitoring step of the planning process because there was no discussion with Annie about the monitoring of her investments or next steps.
    C. John may have breached the presentation step of the planning process because emailing a list of funds does not meet the necessary presentation guidelines.
    D. Assuming John followed all FINRA and SEC guidelines regarding working with international investors, this is a suitable method of implementing Annie’s plan.
A

C. John may have breached the presentation step of the planning process because emailing a list of funds does not meet the necessary presentation guidelines.
Feedback:
Answers A and B, although possibly true, are not the root cause of the issue here. The main problem John has created here is that in doing this favor for Annie, he has not met the guidelines of presenting a plan set forth by the CFP® Board which requires that the CFP® practitioner shall communicate the factors critical to the client’s understanding of the recommendations. These factors may include but are not limited to material: personal and economic assumptions; interdependence of recommendations; advantages and disadvantages; risks; and/or time sensitivity. Sending a list only of the recommendations clearly does not explain these factors to Annie.

178
Q
  1. What is the major difference between a red herring and a prospectus?
    HINT: Think simple if you do not know. The question is quite easy if you eliminate the wrong answers.
    Response:
    A. A red herring is a communist stock.
    B. A red herring omits the selling price and the size of the issue.
    C. If the front page of the prospectus is printed in red, then it is a red herring.
    D. The SEC does not have to approve a red herring.
A

B. A red herring omits the selling price and the size of the issue.

Feedback:
Although Answer C is true, the major difference is Answer B. The SEC has to approve a red herring. Yes, questions come from material not covered.

179
Q

Score 0 of 1
26. Carrie Adams, a 45-year-old single mother of three boys, ages 10, 12 and 14, has come to you seeking financial planning advice. Carrie’s husband, Adam, a self-employed general contractor, passed away eleven-months ago in a tragic bulldozer accident. Adam’s business had been fading a bit due to spending more time with his children. His business was a sole proprietorship, subcontracting out labor on various projects. Adam spent the majority of his time and financial resources on growing his business. He was not focused on saving for the future, rather living for the moment. He believed he would be positioned for a huge growth in business, leading him to an early retirement. Unfortunately, that did not happen, and Carrie is stuck to figure things out. On Carrie’s belief that Adam’s death could have been avoided, she hired an attorney to sue the real estate developer for wrongful death. A disgruntled worker for the real estate developer testified as to the company’s lack of safety procedures. This led to a quick decision and Carrie was awarded compensatory damages in the form of a twenty-year period certain annuity paying her $2,500 per month (this was a dollar figure determined by a court hired economist based on a variety of factors).
Adam did not have any life insurance, and the little bit of savings they did have is dwindling by the moment. Carrie has been out of the work force for the last fourteen years, concentrating her time on taking care of the home and homeschooling her children. Her three boys, Andrew, Anthony and Alex, were enrolled in public school shortly after their father passed. Prior to the birth of Andrew, the oldest, Carrie worked as a paralegal. Since Adam’s income was not always stable, Carrie, conservative by nature, always kept her paralegal licenses up to date. As a result, she was able to find full time work quickly and next week Carrie begins work at a mid-size law firm - the same firm that represented Carrie against the real estate developer. Her salary is $75,000 per year. Her employer does provide some benefits to employees. The law firm provides a high deductible HSA group health insurance, although premiums are paid 100% by the employee for themselves and family. Retirement benefits are also provided through a 401(k) plan and a free annual financial plan conducted by Victor Vector, a CFP® professional. With the lawsuit and her job, she is not too worried about paying the bills. Her concerns are (in no particular order), saving for retirement, providing a college education for her children, enrolling the boys into a private school as soon as possible (she doesn’t like public school) and paying off her mortgage.
During your first meeting you discover that Carrie does not have any life insurance, disability insurance or estate planning documents. Left over from Adam’s general business account is $30,000, which she transferred to her personal checking account. She has spent some of it over the last eleven-months for legal fees and to pay monthly living expenses. Carrie was the beneficiary on Adam’s SEP-IRA. The current value is $150,000. She estimates her monthly essential expenses to be $4,500 per month. Given Carrie’s situation, which of the following recommendations would you make as a CERTIFIED FINANCIAL PLANNER™ professional?

I. Purchase a maximum amount of 20-year term life insurance that she could be underwritten for by a top-rated life insurance carrier.
II. Contribute to a Coverdell ESA and their state’s pre-paid 529 plan to allow her to fund current private school tuition and future college cost of tuition.
III. Contribute to her employer’s 401(k) plan with a 50% match up to 12% of her salary (5-year vesting schedule).
IV. Purchase a $400,000 Variable Universal Life Insurance policy, contributing extra premium (non-MEC) to the policy to help with retirement savings.
V. Establish a monthly savings plan to build up her emergency fund to an adequate level.
Response:

A. II, IV, V
B. I, V
C. I, III
D. III, IV, V
E. I, II, V

A

Response:

C. I, III
Feedback:
It is important that Carrie takes care of her three children. In the event of her untimely death, they could be left destitute. This question does not mention anything about a wealthy family or others that could take care of her children in the event she was to pass away. It is vital that Carrie obtain life insurance. Of the choices including life insurance Roman Numeral I, 20-year term life insurance, is a better choice than Roman Numeral IV, Variable Universal Life. Term life insurance will cost her less in monthly premiums and she should be concerned more about her children’s wellbeing than tax deferral and supplemental savings.
Providing maximum death benefits at a lower overall premium is a better option than cash value life insurance, at this point in her life.
Carrie’s gross income is $75,000 per year. Considering her situation single with three children her income tax liability is minimal. Her monthly net income will be roughly $5,500 per month. Considering her essential expenses are $4,500 per month, she does have a little bit of room to save. It is more important for her to save for her retirement than for her children’s education. Both saving for retirement and education are goals of hers, however, saving for retirement is more important at this stage in her life, especially considering the generous 401(k) match. Thus, Roman Numeral Ill is a better choice than Roman Numeral II. Plus, saving 12% of her salary in a 401(k) is $5,400, or a ~$400 net monthly loss of income. Since her essential expenses are $4,500, this still gives her an extra ~$600 per month on discretionary expenses. Finally, her children can always get loans for college. There has yet to be a retirement loan created… yet.
Lastly, Carrie had $30,000 in her personal checking account. Even though she is a single income earner, she does have a second source of income, the 20-year
period certain annuity. An adequate emergency fund would be at least three months of her essential expenses (3 months x $4,500 = $13,500). The best answer
is C.
Suggestion: I would have read the answers first. This is a very long question.

180
Q
  1. Mr. Able created a $1,000,000 GRAT. He reserved a 5% annuity for a period of 20 years. If the grantor retained interest is $563,165. What is the amount of the taxable gift?

Response:
A. $419.835
B. $425,835
C. $436.835
D. $549,165
E. $563,165

A

C. $436.835

Feedback:
The amount of the gift ($1,000,000 gift - 563,165 retained interest) is $436,835. This is a gift of a future interest, not a gift of a present interest. This is why Answer A is wrong ($17,000). The taxable gift is $436,835

181
Q

Well, enrolling in Medicare disqualify somebody from contributing to an HSA?

A

Yes. Enrollment in Medicare A or B will disqualify you.

182
Q
  1. During the year, Fred Smith had the following expenditure for his rental house. The expenditures are listed below. Fred has a full-time job and files a Schedule E (active participation) for the rental house. Which of the expenditures must be depreciated rather than deducted as an expense on his Schedule E?
    I. Replaced a screen in a window
    Il. Replaced the air conditioning system
    III. Built a swimming pool
    V. Installed a new water heater
    V. Hired a lawn service to cut the grass

Response:
A. I, II, V
B. 1, IV, V
C. II, III, IV
D. III, IV
E. III

A

C. II, III, IV
Feedback:

Items I and V are expenses. The others are improvements even though Il says replace and IV says install. There is no list of what are expenses and what are improvements.

183
Q
  1. While on a trip, Sandra’s wedding ring disappeared. The ring was worth $10,000. She has an HO-3 policy. Which one of the following statements is true?

Response:
A. The coverage on the ring is limited to $1,000.
B. The coverage on the ring is limited to dollar amount ($1,000 - 1,500) if the loss is due to theft.
C. If she has more than $10,000 personal property coverage, the ring is covered for its full value.
D. The ring is not covered.

A

B. The coverage on the ring is limited to dollar amount ($1,000 - 1,500) if the loss is due to theft.

Feedback:
The ring is only covered if its disappearance is due to theft. The dollar amount shown is not important. See Insurance prestudy lesson 3. It is a concept question (theft). There is no coverage unless a theft occurred. The theft must be reported to the police. It is a common sense question/answer. Some policies cover jewelry up to $5,000.

184
Q
  1. A client, T.T. Banks, wants to meet with you. His son, Tubby, finally, graduated from college. T.T. wants to establish a brokerage account for his son to encourage him to start earning money and invest it. He will open his son’s account with a $6,500 gift. How should you proceed?

Response:
A. Open the account and place the money in a Roth IRA. Since it is a contribution, Tubby can remove the money tax-free.
B. Request a joint meeting with T.T. and Tubby to start the planning process.
C. Open the account and place the money in a money market account until Tubby gets a job.
D. Require Tubby to come in to complete a cash flow analysis and risk questionnaire before you can open the account do any investments.

A

.
D. Require Tubby to come in to complete a cash flow analysis and risk questionnaire before you can open the account do any investments.

Feedback:
You cannot do a Roth. Tubby has no earned income. Answer C is not bad. Answer D is the first step in the financial planning process. Should T.T. be involved in the process since he is making the gift. Tubby will be the client.

185
Q
  1. Which of the following organizations would be considered public charities (50% organizations)?

I. Education organizations with regular faculty and curriculum and regularly enrolled students
Il. Hospitals
III. The Rotary Club (A not-for-profit), which raises money for public causes
IV. The United Way
V. Public libraries

Response:
A. All of the above
B. I, II, III, IV
C.I, II, IV, V
D. II, IV
E. III, V

A

C.I, II, IV, V
Feedback:
Rotary is a non-profit (not-for-profit). It isn’t a public or private charity. The Rotary Club would have to obtain IRS approval to receive tax deductible contributions.

186
Q
  1. A client has investment income of $50,000. The client takes a loan of $500,000 from a bank to buy 20 acres of land for investment purposes. The real estate will be the collateral for the loan. The loan cost for the year is 10% or $50,000. Is the interest tax deductible?

Response:

A. No, it is a passive investment (passive loss).
B. No, the client is not materially participating in real estate.
C. Yes, up to investment income.
D. Yes, but the client has to have an AGI of at least $50,000.

A

C. Yes, up to investment income.

Feedback:
Investment interest ($50,000) is tax deductible up to investment income ($50,000). Investment interest is any interest incurred to purchase property that is held for investment, such as raw land.

187
Q
  1. Which of the following is/are considered conduit income?

I. K-1 income from an S corporation
II. K-1 income from a partnership
Ill. Dividend income from a corporation
IV. DNI from a trust
V. K-1 loss from active participation in real estate

Response:
A. I, II, III
B. I, II, IV
C. II, III
D. II, V
E. IIII

A

B. I, II, IV

Feedback:
DNI is distributed net income. A simple trust uses the conduit principal (DNI). Answer V would have been correct if it said K-1 income. Dividends from a corporation are after-tax distributions.

188
Q
  1. What is the advantage between taking excessive income as a stockholder from a corporation or an S corporation if the corporation is very profitable?

Response:
A. Taking earned income as compensation from a corporation will qualify the stockholder to ultimately get more Social Security benefits.
B. Taking earned income as compensation from a corporation will qualify the stockholder for a large retirement contribution.
C. Taking limited income from an S corporation will reduce FICA and FUTA taxes because the remainder of income will be unearned income.
Feedback:
D. Taking limited earned income from an S corporation and the remainder as unearned income will reduce corporate taxes.

A

C. Taking limited income from an S corporation will reduce FICA and FUTA taxes because the remainder of income will be unearned income.

Feedback:
In answer A and B there are salary caps that limit what qualifies for Social Security benefits and retirement contributions. But compensation is subject to unlimited Medicare taxes (disadvantages). That is why Answer C is a better answer. Unearned is not subject to Medicare. Answer D is incorrect, an S corporation does not pay corporate taxes (conduit entity).
You may have chosen Answer D because it is an advantage for an S corporation owner. Yes, S owners reduce their wages to pay less FICA taxes by taking the maximum unearned income. This is why many people do an S corporation - avoid FICA taxes. But that is not what Answer D says. You have to answer the question as written.

189
Q
  1. Suzanna York, age 5, was injured on a playground. The playground equipment was found to be defective, and her parents sued the manufacturer on Suzanna’s behalf. Suzanna was awarded a $1,000,000 compensatory structured settlement to be paid out over 40 years. Unfortunately, Suzanna died during the early stages of the settlement process. What would generally happen in regard to the structured settlement?
    Response:

A. It ceases at Suzanna’s death.
B. Payments continue, but they change from being income tax free to being income taxable.
C. The present value of the remaining periodic payments would be included in Suzanna’s gross estate for federal estate tax purposes.
D. Payments would continue to Suzanna’s estate for the remainder of the 40-terms.

A

C. The present value of the remaining periodic payments would be included in Suzanna’s gross estate for federal estate tax purposes.

Feedback:
Although Suzanna’s applicable credit would probably exceed any tentative tax, the present value of the yet unsatisfied payments would be included in her gross estate for federal estate tax purposes. Nothing indicates the settlement ceases at death (Answer A).

190
Q
  1. For a while you have sensed that Edwin Edwards is no longer mentally competent. While monitoring his investment activities, you learn that he is considering giving his power of attorney to an individual with a history of elder abuse. Whom should you call?

Response:

A. The compliance officer (registered principal) at your broker-dealer
B. Edwin’s attorney
C. Edwin’s children
D. His investment advisor

A

B. Edwin’s attorney
Feedback:
Edwin’s attorney enjoys “client privilege” and cannot be forced to discuss Edwin’s condition with third parties. The question does not indicate that his investments are with your broker-dealer. Edwin’s children are not your clients. You do not want to violate Confidentiality in that way. Discussing Edwin’s condition with the investment advisor also breaches Confidentiality. You could also speak with Edwin’s physician but that is not among the answer choices.

191
Q
  1. Lenny was divorced. He has two daughters with his first wife. A few years before his death, he married Marilyn (second wife). Lennie established a trust for Marilyn. The trust provisions gave Marilyn the right to trust income limited to the ascertainable standards of health, education, maintenance, and support (HEMS). The trust agreement also provides Marilyn with a discretionary right to principal, limited to the same HEMS standard, but which had to be preceded by the exhaustion of Marilyn’s other resources. After Marilyn’s death, the remainder of the trust passes to Lenny’s children. What type of trust does it appear that Lenny established?

Response:

A. A qualified terminable interest property (QTIP) trust
B. A qualified domestic trust (QDT)
C. A Bypass trust
D. A power of appointment trust
E. An estate trust

A

C. A Bypass trust
Feedback:
The provisions for Marilyn reflect a bypass trust (B trust). The right to income limited to HEMS is not a right to all income from the trust as would apply with a QTIP trust. Also, the QTIP cannot use the lifetime exclusion.

192
Q

Are taxable gifts included in the taxable estate?
A. No
B. Yes
C. Maybe
D. Only if the amount of the gifts exceeded the annual gift tax exclusion that was available for the year in which the gift was completed.

A

A. No

Question 3: A
Taxable Gifts are not part of the Taxable Estate. Instead, Taxable Gifts are added to the Taxable Estate to get to the Tax Base.

193
Q

Which of the following powers held by the income beneficiary is (are) considered to be a general power of appointment, thus causing all the trust corpus to be included in the beneficiary’s estate for federal estate tax purposes?

I. The power that can be exercised in favor of the holder for the holder’s health, education, maintenance,
and well-being.
II. The power that can be exercised in favor of the holder subject to an ascertainable standard.

A. I only
B. Il only
C. Both I and lI
D. Neither I nor lI

A

Question 10: A
Well-being is not an ascertainable standard; therefore, it is a general power of appointment. A general power is included in the gross estate.

194
Q

Which of the following will be included in the gross estate of the grantor?

I. Pour-over will assets
II. The present value of the survivor benefit from a joint life and survivor annuity
III. A gift to a charitable remainder trust (CRT) at death for the benefit of the grantor’s spouse
IV. Assets in a funded revocable trust
V. Assets in a funded irrevocable trust

A. All of the above
B. I, II, III, IV
C. I, II
D. I, III, IV
E. II, IV
II, IV

A

Question 2: B
A pour-over will normally passes the decedent’s separately owned assets into a trust. A gift at death is included in the gross estate even if it is made to a charity or to a spouse. The CRAT is included in the gross estate because the spouse has an interest. Then it ultimately passes tax-free to the spouse by the marital deduction. When she dies it passes to charity. The annuity is included. Chapter 4 addresses how an annuity is included in the estate.

195
Q

Distributable net income, DNI, is a concept that has been developed for which of the following purposes except:

A. Limiting the amount of distributions that may be taxable to the beneficiaries.
B. Advising beneficiaries of the amount of income the trust has earned that represents their interest.
C. Establishing the character of the amount taxable to the beneficiaries.
D. Limiting the deduction, a trust may receive for amounts distributed to beneficiaries.

A

Question 5: B
DNI accounts for the income to the beneficiary as well as the corresponding deduction for the trust.

196
Q

Question 11
John loves his 12-year-old daughter. He and his wife want to give her a gift in a UTMA account for college education purposes. Which of the following investment strategies would be most appropriate if John is in the 35% marginal tax bracket?

A. Preferred stock paying a 6% dividend worth $30,000
B. A 5-year CD ($30,000) with a 6% rate
C. Waiting until his daughter turns age 24
D. A corporate bond paying 7% worth $30,000

A

B. A 5-year CD ($30,000) with a 6% rate

The $1,800 is under the level of unearned income where the kiddie tax starts ($2,500). Also consider the investment time horizon. His daughter will be going to school in approximately 5 years when the CD will mature for face value. John should not wait until his daughter is age 24. This is a favorable tax situation. The corporate bond ($2,100) and the preferred stock are exposed to interest rate risk.

197
Q

Question 13
Bill opens an account for his 16-year-old son, Jim. Bill is concerned about Jim’s spending habits. Jim may be going to college, but that isn’t certain. Bill doesn’t mind Jim getting income from the account in the future.
Which of the following arrangements best satisfies Bill’s objectives?

A. An UGMA
B. An UTMA
C. Section 2503(b) trust
D. An irrevocable trust with Crummey provisions

A

Question 13: D
With a 2503(b) trust, principal is not required to be distributed. However, income distributions are mandatory. The gifts to the trust will mainly be gifts of a future interest. The irrevocable trust can solve all the concerns, but the son will have to waive his Crummey rights.

198
Q

Which of the following charitable transfers allow for distributions to be paid to a life annuity or for a term certain (up to 20 years)?

I. CRAT
II. CRUT
IlI. Pooled income fund
IV. Gift annuity

A. All of the above
B. I, II, III
C. I, II
D. II, III
E. III, IV

A

Question 4: C
Pooled income funds and gift annuities do not allow distributions of income as a term certain other than the actual life of the beneficiary.

199
Q

Question 3
Lester Landon would like to gift $517,000 (FMV) of appreciated property (basis $200,000) to his son. Lester doesn’t want to use his liquid assets to pay the 40% gift tax. He already has gifted his full lifetime exemption.

What gifting technique makes the most sense for Lester?

A. Do not make the gift
B. Make a net gift and have the son pay the gift tax of $142,857
C. Make a net gift and have the son pay the gift tax of $147, 143
D. Have the son pay the gift tax of $200,000 using a reverse gift technique
E. Use the annual exclusions only for the gift of the property

A

B. Make a net gift and have the son pay the gift tax of $142,857

The normal gift tax would be $200,000 ($500,000 x 40%).

However, when the son pays the gift tax for this net gift, the amount will be $142,857 ($200,000 ÷ 1.40). The 1.40 is 1 plus the gift tax bracket (40%).

200
Q

Which of the following circumstances would cause the date-of-death value of gifted property to be included in the donor’s gross estate?

I. The donor retains a life estate in the gift property.
II. The donor retains the power to revoke or amend the gift.
III. The donor gives more than $17,000 to one done in one year.
IV. The donor dies within 3 years of the date of the gift.

A. I, II, III
B. I, II
C. II, IV
D. III, IV
E. I, II, III, IV

A

Question 4: B
Transfers with retained interest are brought back into the gross estate. Ill would not cause the date-of-death value of the gifted property to be included in the donor’s gross estate. It is added to the taxable estate. IV is debatable. For example, life insurance would be included at date-of-death value if given within 3 years of death. IV is too broad an answer to be correct. Except for 529 plans, transferred property where the transferor holds a retained interest of any kind is included in the gross estate.

201
Q

John, age 38, and Mary Reed, age 35, have limited assets but a promising future. Other than a home (with a substantial mortgage balance), IRA and 401(k) accounts (limited), and some personal property, they have some cash and investments. They each individually have a $2,000,000 life policy naming each other as primary beneficiary and their two children (8 and 10) as secondary beneficiaries. Which is the best beneficiary designation for their life insurance policies?

A. Leave the beneficiaries unchanged
B. Change the primary beneficiary to the children
С. Set up revocable living trusts with exemption provisions and name the trust as the beneficiary of the policies
D. Set up testamentary trusts with exemption provisions leaving the spouse as the primary beneficiary and naming the trust as the contingent beneficiary

A

Question 5: D
Rather than leaving the policies in trust, when one spouse dies, the survivor should get all the assets (Answer D). Answer D is arguably the more practical answer with the current $12,920,000 exemption or $25,840,000 together. Answers A and B won’t work because the children are minors. The Reeds have limited assets except for life insurance. If the revocable trust is the named beneficiary, the life insurance ($2,000,000) and all other separate assets could go into the applicable credit amount trust at the first death. These assets could remain there for 40-50 years.

202
Q

Which of the following planning techniques are considered “freezing” techniques for transfer tax purposes?

I. A GRAT
Il. A preferred stock recapitalization (recap)
III. A QPRT
IV. A private annuity
V. A SCIN

A. All of the above
B. I, II, III
C. II, III
D. IV,V
E. IV

A

Question 4: A
With a GRAT and A QPRT, the value of the transferred assets are frozen when transfer is made and a taxable gift occurs. To keep the property out of the grantor’s gross estate, the donor must outlive the term. A recap is a freezing technique using preferred stock. With a private annuity or a SCIN, the property is transferred for a stream of income, and if the owner dies, no value is included in the estate.

203
Q

Which of the following statements concerning a Grantor-Retained Annuity Trust is incorrect?

A. A GRAT generally provides a fixed income for the grantor’s life.
B. The property transferred to the trust has a string on it.
C. The gift portion of the GRAT transfer is a future interest gift to the remainderman.
D. The GRAT only saves estate taxes only if the grantor lives beyond the trust term.

A

Question 8: A
The GRAT has a fixed income for a number of years.

204
Q

Question 9
Frank gifts the following money and assets to various family members as shown below:

-$34,000 cash to Purdue University for his nephew’s college tuition (split gift)
-$40,000 cash to his daughter to purchase a new car (split gift)
-$50,000 cash to his son for the down payment on a home (split gift)
-A life insurance policy with a death benefit of $1,000,000 and a cash value of $52,000 to his wife
-Stock with an original basis of $10,000, now worth $50,000 to his mother (split gift)

Do Frank and his wife have to file a federal gift tax return (Form 709)?

A. Frank must file a gift tax return for $19,000 (taxable)
B. Frank must file a gift tax return for $22,000 (taxable)
C. Frank must file a gift tax return for $44,000 (taxable)
D. Frank and his wife must both file gift tax returns of $19,500 each (taxable)
E. Frank and his wife must both file gift tax returns of $22,000 each (taxable)

A
205
Q

Question 15
Which of the following trusts could include ascertainable standard provisions (HEMS) for a widow or widower?

I. This trust holds assets (income & corpus) intended for the sole use, enjoyment or benefit of the surviving spouse.
II. This trust will provide the surviving spouse with a stream of income that will be paid for life yet also qualifies the property for the marital deductions
III. This trust property qualifies for the marital deduction but is structured so that the surviving spouse does not receive corpus during his/her lifetime and could receive income, if available, from the trust at the discretion of the trustee.
IV. An irrevocable trust is established to receive assets that are disclaimed by the surviving spouse of a decedent.

A. I, II, IV
B. II, IV
C. I, III, IV
D. III

A

Question 15: D

Trust Il is a QTIP(c) trust that may enable the widower) to receive principal per HEMS. Trust IV A Disclaimer Trust (like a non-marital) enables the surviving spouse to receive income, but distributions will be subject to an ascertainable standard (special could have HEMS)

206
Q

Question 21
Grandpa gave you $2 million in low/no basis stock paying a 3% dividend. To maintain your standard of living, you need a 7% return for life. Which of the following charitable gifting techniques could meet your requirements?

A. Charitable gift annuity
B. Annuity Trust (CRAT)
C. Unitrust (CRUT)
D. Pooled Income Fund
E. Charitable stock bailout

A

Question 21: B
A CRAT must pay out a fixed percentage of the initial FMV (at least 5%). The specified amount must be paid at least annually to the non-charitable beneficiary out of income and/or principal. With a CRAT, capital gains on the low basis stock will be avoided. With a charitable gift annuity, only part of the capital gains is avoided. With a CRUT, a fixed percentage of the net FMV, as revalued annually, must be payable. 7% return is not assured.

207
Q

Question 24
Lamar is considering a program of lifetime giving. He plans to set up irrevocable trusts for the benefit of his 2 grandchildren. Alice will not consent to gift splitting with Lamar. She plans to gift separately. If Lamar gifts the following amounts over the next 5 years, what will be the amount of taxable gift?

Gift Made:
- 2024 — $65,000
- 2025 — $77,000
- 2026 — $91,000
- 2027 — $104,000
- 2028 — $117,000

A. $85,000
B. $170,000
C. $370,000
D. $455,000

A

Question 24: D
The gifts are of a future interest and don’t qualify for the annual exclusion [not $455,000 - $85,000)]. NOTE:
Just because it is an irrevocable trust does not mean it has Crummey provisions. This is not a gift to a 2503(c) trust nor did the irrevocable trust provide Crummey provisions.

208
Q

Question 28
Lamar purchased a custom van to be used in his business. He requested coverage from his auto insurance (property/casualty) agent within (60) days of purchase. The van is a substitute for his personal auto. Will the van be covered under his current auto policy?

A. The van will be covered under his PAP.
B. The van will not be covered because he waited beyond the (30) day period.
C. The van will not be covered because it is used in business.
D. The van will be covered because Lamar called and requested coverage as a substitute for his personal auto.
E. The van will be covered because it will be owned by Lamar and used as a private passenger automobile.

A

Question 28: C

Vans used in business cannot be covered by a personal auto policy; they have to be covered under a business policy

209
Q

Question 29
Lamar and Alice need estate planning advice regarding gifting. Which of the following statements are true?

I. The federal gift tax applies to all gratuitous transters.
II. “Gift splitting” means that spouses may file a joint gift tax return.
III. If all the gifts of a donor in a given calendar year are present interests, a federal gift tax return is not required unless the gifts to any one donee exceed $17,000 in value.
IV. If Lamar pays $17,000 for tuition to Chesapeake County Day School for his niece, this payment will result in a taxable gift of zero.
V. The $12,920,000 exemption is not elective; it must be used if available for gifts.

A. I, II, V
B. II, III, IV
C. III, IV, V
D. III, V
E. IV, V

A

Question 29: C

I is wrong because all gratuitous transfers are not subject to federal gift taxes. Examples include political contributions, payments of tuition directly to an educational institution, and direct payment of another person’s medical and hospital bills. Il is false because spouses must file a gift tax return if one spouse consents to “gift splitting.” IV is a direct payment of tuition. The exemption must be used if gifts exceed $12,920,000, then gift tax is payable at 40%.

210
Q

Question 50
Currently, Cynthia has $40,000 in her UTMA account. Tom feels that Cynthia will need $100,000 to complete four years at college. Which of the following techniques would work best considering the facts in the case?
Hint: From question 43, she is paid $7,240 per year.

A. Pay Cynthia more per hour
B. Gift to Cynthia $30,000 per year over the next four years
C. Enter into a gift leaseback with some of Sun City’s office equipment
D. Rollover the $40,000 in Cynthia’s UTMA to a 529 plan and in addition gift $15,000 per year to the plan
E. Leave the UTMA alone and start gifting $15,000 per year to a 529 plan

A

Question 50: E

Using $40,000 as a PV, an 5% return and 4 years, the payments (end), Tom needs to gift only a total of $15,000 per year to a 529 plan to fund the $100,000. The UTMA is partially funded by Cynthia’s earnings. That should retain her money. Tom can recover money from the 529 plan, but it should be his money. The gift lease-back would be paid with tax deductible dollars. However, this is unearned income and would be subject to the kiddie tax.

211
Q

Question 52
The divorce settlement requires the life insurance policy to remain in force for 20 more years (until his ex-wife is
65). Which of the following is the best tax alternative for Tom?

A. Transfer the existing policy to his ex-wife
B. Buy a 20-year term policy on his own life
C. Buy a new universal life policy on his own life
D. Let his ex-wife purchase a policy on his life (He pays premium.)
E. Do nothing for the next 20 years and take a chance he will not die within the timeframe

A

Question 52: D
If Tom transfers the current policy to his ex-wife, he will have to get his current wife to consent to gift-splitting and file two gift tax returns. In addition, the policy will remain exposed to inclusion in his gross estate for 3 years. Answer B will accomplish nothing. The 20-year term policy remains in his estate and the premium is not tax deductible. If his ex-wife purchases a policy on his life, it is not included in his estate, and the premium can qualify as alimony (income tax deductible).

212
Q

Question 54
R.J. wants to set up a trust for his problem child, Ashley. He wants to contribute money yearly to the trust, but Ashley would only get income (no 5 or 5 right). The residue would go to his grandchildren at her death (her children). Ashley would receive all the earnings from the trust no matter how much it made. He realizes he will have to give the money away irrevocably but is desperate to get her on the right track. Which of the following trusts would be most effective?

A. 2503(c)
B. 2503(b)
C. A Support Trust
D. A Grantor Trust
E. An Irrevocable Trust with Crummey provisions

A

Question 54:
The 2503(c) trust is for minors. The 2503(b) will work, but yearly gifts would be gifts of a future interest and won’t qualify for the annual exclusion. A support trust is established to discharge an obligation. R.J. doesn’t have an obligation to a grown child. A grantor trust would cause the income to be taxed to R.J. With regard to an irrevocable trust, please go back to Chapter 6. Yes, she could demand under the Crummey provisions, but none of the other trusts work.

213
Q

Question 51
Tom owns a life insurance policy that was required under of the divorce settlement. When he dies, how will the policy affect estate taxes at his death?

A. The policy was part of a divorce settlement and will not be included in his estate.
B. It will have no effect on estate taxes at his death.
C. It will use $250,000 of Tom’s applicable exemption amount.
D. It will use $10,000 (CV) of Tom’s applicable exemption amount.
E. Only the remaining amount of alimony payments will affect Tom’s exemption amount, not the death
benefit of the policy.

A

Question 51: C
Because Tom owns the policy it is included in his estate. If the insured possessed any incidents of ownership at death, exercisable either alone OR IN CONJUCTION WITH ANY PERSON (such as the irrevocable beneficiary), then it’s included in the insured’s estate. With his ex-wife as beneficiary, he will have to use $250,000 of his exemption amount which will diminish the amount he can transfer into the bypass trust.

214
Q

Question:
2. Your client, Alice owns the following four different diversified mutual funds:
Growth fund
Emerging market fund
Government bond fund
Corporate bond fund
$45,000
$14,000
$50,000
$35,000
Alice is concerned about overall portfolio risk. She is concerned about standard deviation and other factors. Due to a recent inheritance, she has additional money to invest. To which among her currently held mutual funds do you suggest she add money?

Response:
A. The growth fund
B. The emerging market fund
C. The government bond fund (Treasury securities)
D. The corporate bond fund

A

B. The emerging market fund
Feedback:
The emerging market fund currently represents the smallest percentage of the portfolio allocation and likely has the lowest correlation coefficient relative to the other funds. Reducing correlation coefficient would reduce the portfolio’s overall risk. The correlation coefficient is highly important for the exam.
(Somewhat subjective)

215
Q

Question: 3. When, if ever, can a corporation that issues qualified stock options (ISOs) receive a tax deduction for the ISOs?

Response:
A. Never
B. Always
C. Yes, if the ISO is disqualified
D. Yes, if the ISO is qualified
E. Yes, if no more than $100,000 worth of ISO stock is granted that vests in a specific year

A

C. Yes, if the ISO is disqualified

Feedback:
If the stock that was acquired under the option (right to buy) is sold before the two year /one year holding period, the excess of the fair market value of the shares at the time of exercise over the exercise price is treated as compensation to the option holder. That creates a corresponding deduction for the issuing corporation.

216
Q

Question:
6. Following the death of the grantor (trust maker) which of the following strategies should be the most effective to reduce GSTT?

Response:
A. An irrevocable trust
B. A revocable trust
C. A reverse QTIP
D. A dynasty trust

A

C. A reverse QTIP
Feedback:
The decedent can still use the GSTT exemption when the reverse QTIP is elected by the executor (after death). The dynasty trust is generally implemented before death.

217
Q

Question: 12.
You are a CFP ® certificant. Todd and Belinda Harding are you new clients. During the initial interview Todd excuses himself for a restroom break. Belinda whispers to you that Todd is a compulsive gambler. She confides in you that she has managed to squirrel away a significant amount of cash that, at the moment, is in a money market account. Belinda asks you not to tell Todd about the account and says that she wants to call you the following morning to discuss allocation options for the money. How should you best handle this awkward situation?

Response:
A. When Todd returns to the room and as you begin to gather data, act as if you have no awareness of the account that Belinda mentioned.
B. Speak to Belinda the following morning to discuss her allocation choices.
C. Terminate the relationship before you proceed to the data gathering step of the financial planning process.
D. When Todd returns from the bathroom, tell him about the account.

A

Feedback:
Who is the client? The presumption was that both Todd and Belinda would become your clients (not just Belinda). This is the initial interview (going to be clients). Regarding Answer D, Belinda makes clear that she does not want her compulsove gambler husband to know about the account. Further, without the disclosure of the value of the assets, including the money market account, the data is too vague for meaningful financial planning

218
Q
  1. Your client, Bob Hill, is still bristling over his divorce in 2018. Under the divorce agreement, he made alimony payments to his ex-wife of $50,000 in the first post-separation year, $20,000 in the second year, and nothing in the third year. Considering the Internal Revenue Code, which of the following statements can you accurately explain to Bob?

Response:
A. The payments to date do not qualify as alimony. Bob needed to pay his ex-wife for at least three full years.
B. Under the divorce decree, of the payment of $50,000, only the $20,000 would be treated as alimony. The remaining $30,000 would be regarded as a property settlement.
C. Bob would be required to recapture $32,500 of what he has already paid to his ex-wife and report it as income at the end of the third year. His wife would probably file amended returns as well.
D. If part of the first $50,000 payment went directly to a building contractor to remodel the ex-Mrs. Hill’s new condo, that portion would be treated as part of the property settlement rather than alimony.
E. Bob would be required to recapture $20,000 of the first-year payment and would not be eligible for any deduction in the second year.

A

C. Bob would be required to recapture $32,500 of what he has already paid to his ex-wife and report it as income at the end of the third year. His wife would probably file amended returns as well.
Feedback:
The alimony appears to be front-loaded and thus subject to recapture. Both Bob and his ex-wife will file ammended returns.
Payment year #1 and Year #2
$70,000
minus constant
=37,500
Recapture
$32,500

219
Q

Score 0 of 1
Question:
17. Smokestack, Inc. voluntarily terminated its defined benefit plan. Your client, Homer Connors, age 61, has been a long-term employee of Smokestack, Inc. and a participant in this pension. The
“termination” has made Homer quite anxious. What might you tell Homer that may make him feel less anxious?

Response:
A. The 10% penalty (59-1/2 year rule) will not apply to distributions.
B. The account balance must be rolled over into an IRA account.
C. Homer is 100% vested.
D. The plan is fully funded. There is no need to worry.

A

C. Homer is 100% vested.
Feedback:
The 10% penalty will not be imposed on Homer because he is over 59½ and is a possible answer. The plan is fully funded at normal retirement age, not necessarily at a premature termination. Homer would get the account balance that is attributable to him and fully vested.

220
Q

Question:
30. The following irrevocable trusts are producing annual income. From which of the trusts below will the income not be taxable to the grantor?
Response:
A. Trust income is retained to discharge a legal obligation of the grantor.
B. Trust income is retained then used to pay premiums on life insurance on the life of the grantor.
C. Trust income is distributed to the grantor for 5 years, and then distributed to another trust beneficiary.
D. Trust income is distributed to the grantor until death, and then distributed to the other trust beneficiary(s).
E. Trust income is accumulated for later distribution to the other trust beneficiary(s).

A

E. Trust income is accumulated for later distribution to the other trust beneficiary(s).
Feedback:
The income will be taxed to the trust. The grantor has no beneficial enjoyment. The other answers violate the grantor trust rules that make the income taxable to the grantor.

221
Q
  1. Sally donates several bags of old clothes to the Salvation Army. Which statement below best reflects the documentation that Sally would need in order to claim a charitable income tax deduction?
    Response:

A. Deduction of up to $250 does not require a receipt.
B. Deduction of $250 but less than $1,000 must be documented.
C. The deduction is the lesser of fair market value or the donor’s basis (substantiated).
D. The deduction is limited to basis (unsubstantiated).

A

C. The deduction is the lesser of fair market value or the donor’s basis (substantiated).

Feedback:
For charitible gifts less than $250, a dated receipt is proof for purposes of an income tax deduction. The receipt should include a description of the property. A written receipt would list the items donated with a corresponding value. Sally should keep records showing the fair market value and her cost basis. For charitable gifts of $250 or more, Sally must substantiate the deduction by written acknowledgement from the charity. Cash donations up to $300 single/$600 joint do not have to be documented for 2021 if you take the standard deduction

222
Q
  1. A CFP® professional meets a prospective client who is prepared to discuss his retirement accounts that were provided through a former employer. The client is concerned that the investments in those accounts are too aggressive. He states that he and his wife would like to retire when they are 65 and that they have recently been writing many checks from their account to support their adult son who can’t seem to find a job. After further analysis, the CFP® professional determines that the client does not have enough cash flow to retire when they are 65 while continuing to support their son. How should the CFP® professional proceed?

Response:
A. Communicate to the clients that their retirement goals are unrealistic and that they should plan to work until age 70, and that they should stop giving money to their son.
B. Help the clients to review and prioritize their goals.
C. Review the client’s current and potential income streams to identify ways to solve the immediate cash flow shortfall.
D. Recommend a conservative asset allocation model to reduce the risk in his retirement accounts and suggest that they stop giving money to their son.

A

B. Help the clients to review and prioritize their goals.

Feedback:
The CFP®Board would feel that it is important to encourage the client to reassess priorities before making specific recommendations. Answer C is wrong because it assumes the client has established new goals. Answers A and D are arguable.

223
Q
  1. You have been advising Amy and Cal Campbell for years. At a monitoring meeting in your office, they sit as far apart as possible. They seem to be avoiding eye contact with each other. Amy speaks up first. She tells you that they have been in marriage counseling for 6 months without progress. She wants to retain you as an advisor. Cal glares at her and says that you must make a decision as to which one of them you will work with. What would be your most appropriate response in this situation?

Response:
A. Terminate the relationship
B. Understand that they may divorce. However, until they are divorced you may represent both of them.
C. Have them sign separate contracts with you.
D. Recommend that each of them interview other advisors. That way each can receive separate financial counseling.

A

B. Understand that they may divorce. However, until they are divorced you may represent both of them.

Feedback:
At this point, you represent both spouses. After the divorce and the assets are split you could represent both of them with separate contracts.

224
Q
  1. Your client Clara Bow, age 80, has enjoyed a full life. Oscar, her husband of 60 years recently died. Because he had several serious health issues, his death was not unexpected. During the last years of Oscar’s life, they traveled extensively and stayed at 5-star hotels. Oscar was an inventor. His patent royalties produce more income than she can spend. Clara’s charitable wishes focus on the church she has attended throughout her lifetime. She was both baptized and married there. Clara’s family includes two married children plus 4 sets of married grandchildren. Oscar did extensive investing with you and after he died you continued to advise Clara regarding investments and financial matters generally. Before he died, Oscar told you that many of the family members relied on gifts and loans from their parents. Oscar had been frustrated that his children and grandchildren had not been more self-reliant. Now Clara wants advice from you as to gifting to family members. What would you recommend to Clara?
    Response:
    A. Limited gifting to the excess money she is not spending.

B. Clearly understand the annual gift tax exclusion limit, the overall limit on tax-free lifetime giving and how the Form 709 works.
C. Gather more data including a current and detailed Statement of Cash Flows. Design and implement uniform gifting allocation to all family members.
D. Explain to her the $12,920,000 gift tax exemption and how it is factored into transfer tax on Form 706.

A

C. Gather more data including a current and detailed Statement of Cash Flows. Design and implement uniform gifting allocation to all family members.

Feedback:
A gifting strategy should be proceeded by careful cash flow analysis. Clara may have health problems, where the excess cash flow would be needed. There is no indication of Clara’s net worth or how long the patent rights will continue. Given her age, Clara may need some of her income for health related and other reasons.

225
Q

Score 0 of 1
Question:
72. Jane Wilson and Gloria Vanderbilt established a corporation many years ago. Unfortunately, Gloria has been battling metastatic breast cancer and has a shortened life expectancy. Gloria is working part-time and spending as much time traveling with family members as possible. Jane is seriously concerned because she does not want Gloria’s husband (who made a pass at her) to become an owner. Jane realizes that purchasing life insurance on Gloria to fund for a buy-out or buy sell agreement is now impossible. What kind of arrangement would you recommend?

Response:
A. Establish a stock redemption buy sell agreement using future corporate profits to buy Gloria’s interest over 10 years.
B. Establish a stock redemption buy sell agreement with Jane agreeing to purchase Gloria’s stock in installments over 10 years.
C. Establish a cross-purchase buy sell agreement with the corporation using future profits to buy Gloria’s interest over 10 years.
D. Establish a cross-purchase buy sell agreement with Jane agreeing to purchase the stock on an installment basis over 10 years.

A

D. Establish a cross-purchase buy sell agreement with Jane agreeing to purchase the stock on an installment basis over 10 years.
Feedback:
The cross-purchase arrangement enables Jane to enjoy stepped up basis. Her basis would not step up under the entity/stock redemption arrangement. In Answer B, Jane does not buy the stock, the corporation buys the stock. In Answer C, the corporation does not buy the stock, Jane buys the stock.

226
Q
  1. Doc Holiday owns a dude ranch that operates as an S corporation. He employs about 5-6 full-time and 6-7 part-time employees. His business includes a bed-and-breakfast operation. He operates as Holiday, Inc. as S corporation. One employee, Jesse, not only breaks in the wild horses but is a wiz at picking the right horse for each dude ranch guest. He cannot afford to lose Jesse as an employee. Other than health insurance Doc provides no other benefits for employees. Doc only takes $2,000 a month as salary and the profit comes out to him as K-1 income. He wants a recommendation from you as to what kind of benefit should he offer to Jesse. What would you suggest?

Response:
A. Establish a non-qualified deferred compensation plan for Jesse using a variable annuity.
B. Establish a 162 “double bonus” arrangement using a fixed annuity or life insurance.
C. Establish a SEP presuming that the 3 out of 5 years of service eligibility rule would eliminate most of the employees.
D. Establish a nonqualified deferred compensation arrangement that would operate through a secular trust.

A

B. Establish a 162 “double bonus” arrangement using a fixed annuity or life insurance.

Feedback:
With the 162 double bonus arrangement, the first bonus would go into the annuity/life insurance and the second would pay Jesse’s phantom tax. Due to its conduit taxation an S corporation cannot offer a non-qualified deferred compensation plan. Since we know little about how long other employees have worked for Doc, it is uncertain as to whether the SEP would eliminate most of the full and part-time employees. The deferred compensation arrangement through a secular trust would create undesirable phantom income for Jesse.

227
Q
  1. Jack and Jill Jones are divorcing. In exchange for her release of all marital rights that she has or may have in his estate, Jack understands that he will, under the terms of the divorce decree, give Jill $1,000,000 as a lump-sum settlement. Under this circumstance, is the $1,000,000 subject to federal gift tax?

Response:
A. No
B. No, if the payment occurs within a 3-year period after the divorce
C. Yes
D. Yes, if it is a gift of a future interest
E. Yes, if it is required under the divorce decree

A

B. No, if the payment occurs within a 3-year period after the divorce

Feedback:
subject to federal gift tax.
The $1,000,000 transfer is not subject to gift tax if the transfer of funds occurs within three years of the date of the divorce decree. It is still regarded as a marital gift and thus it is not

228
Q

Question:
Multiple choice question
19. Jim Harrison, President of Harrison Office Supply and Furniture, Inc. wants to establish a profit-sharing 401(k) plan. He would like to exclude some of the sales staff but is uncertain if qualified plan rules permit this. Jim would like to cover all W-2 (salaried) employees. The office staff consists of 17 W-2 employees (4 are classified as highly compensated and 3 are key employees). The warehouse staff consists of 38 W-2 employees, none of whom are highly compensated. The sales department consists of 6 1099 commissioned salespeople (all on straight commission and all have complete control over their schedule and duties) and 1 W-2 employee, Charles Porter, the sales manager is a highly compensated key employee. Porter receives a salary. bonuses, and commissions on his own sales. Which of the following statements are correct regarding permissible coverage and includable compensation that would generally apply to the 401(k) plan that Jim is considering?
I. If the salespeople are deemed to be common-law employees, the plan may exclude them provided either the ratio percentage test or the average benefit test are passed to indicate that the plan is not discriminatory.
Il. If the salespeople are independent contractors, their participation could cause the plan to be disqualified.
III. Commissions may be excluded from includable compensation for elective deferrals and employer contribution purposes.
IV. Bonuses may be excluded from includable compensation for elective deferral and employer contribution purposes.

A. I, II
В. I, II, III
C. I, III, IV
D. All the above

A

D. All the above

Feedback:
Participation eligibility in a qualified plan is contingent on employee status. (Sole-proprietors and partners may establish their own qualified plan.) An Independent contractor may participate in his own qualified plan but may not participate in another entity’s qualified plan unless he is also deemed to be a regular or common law employee of that entity. It is possible to exclude commissions and bonuses from includable compensation provided that doing so does not discriminate against a non-highly compensated participant.
For example, if commissions are not counted as includable compensation and the key employees never receive commissions, the non-key employees have been discriminated against because their includable compensation is being reduced (by the amount of commissions earned) while the key employees compensation is not reduced at all. Very involved question and answer

229
Q
  1. Harry Stonewall owns HS, Inc. HS is a small manufacturer of perforated metal products sold to larger manufacturers. The cash flow of HS, Inc. varies from month-to-month. Harvey feels that he can train an employee to run a piece of equipment in an hour or so. At times, Harvey fires employees for various legitimate reasons. Most employees stay with HS, Inc. a month or two as a result of firing and layoffs. Harvey is considering the installation of a retirement plan for his company. What would you recommend?

A. A defined benefit pension plan
B. A profit-sharing plan
C. A SEP
D. A SIMPLE
E. A SIMPLE 401(k)

A

C. A SEP

Feedback:
Without knowing Harry’s age or salary the SEP makes the most sense. ERISA only requires 1,000 hours to count as a year of service. But, SEP also says 3 out of the 5 years. No employee will ever make it to 3 years. SIMPLE (answers D and E) are too restrictive and require a match.

230
Q
  1. Marty and Libby Joseph are married and have no children. Marty is a Canadian citizen. He lives full-time in the U.S. and has no wish to become an American citizen.

After Libby dies, he doesn’t plan on going back to Canada. Currently, Marty has about $4 million invested in real estate in Canada in his own name. Libby works as the Chief Financial Officer of a micro-cap corporation making $500,000 to $600,000 a year. Libby has completed her estate plan by establishing a QDT for Marty should she die first. If Libby dies with an estate of $17,920,000 how much should go into the trust?

A. $5,000,000
B. $12,920,000
C. $17,920,000
D. $0, because their joint exemptions protect the entire amount from federal estate tax

A

A. $5,000,000

Feedback:
$12,920,000 will go to Marty tax free. He gets the exemption amount. The other $5,000,000 would by-pass estate taxes until he dies. If all goes into the QDT she would lose her exemption at her death. This is similar to a question in the Live Review book.

231
Q

Multiple choice question
42. Which of the following statements is correct concerning income in respect of a decedent (IRD)?

A. If the income is included in the gross estate, the estate tax attributable to that income item is generally deductible by the recipient of the income.
B. The items of income will only be subject to income tax, not estate tax. Double taxation is avoided.
C. Income that the decedent was entitled to receive but had not yet received as of the date of death is excludible in his or her gross estate.
D. An income tax refund not yet constructively received is treated as IRD.

A

A. If the income is included in the gross estate, the estate tax attributable to that income item is generally deductible by the recipient of the income.

Feedback:
Answer B is incorrect because of Answer A. An income tax refund is not IRD.

232
Q
  1. Paul Parker bought a whole life insurance policy 10 years ago. Through an oversight he had a limited pay policy and paid two full premiums in one year. As a result, the policy became a MEC.
    The insurance company pays dividends each year. Which of the following dividend options would not produce an income tax liability to Paul?

A. The dividends accumulate with interest in an account for Paul that is maintained by the insurance company.
B. Paul receives the dividends in cash.
C. The dividends are used to buy paid-up additions to the policy because Paul is concerned about loss of buying power on the death benefit.
D. The dividends are used to reduce upcoming premiums so Paul can use his cash for other purposes.

A

C. The dividends are used to buy paid-up additions to the policy because Paul is concerned about loss of buying power on the death benefit.

Feedback:
Answer A will produce taxable interest. Answer B and D are taxable income under the MEC rules.

233
Q
  1. Regarding ABLE 529 arrangements, which of the following statements is accurate?

A. A disadvantage to the ABLE account is that its beneficiary is likely to lose eligibility for public benefits such as Medicaid.
B. The maximum annual contribution to an ABLE account is pegged to the amount of the annual gift tax exclusion.
C. The maximum annual contribution to an ABLE account is pegged to the annual elective deferral limit for 401(k) plans.
D. Contributions to an ABLE account may not be made for any beneficiary who is older than age 26.

A

B. The maximum annual contribution to an ABLE account is pegged to the amount of the annual gift tax exclusion.

Answer A is incorrect as amounts up to $100,000 do not affect government benefits. Answer D is wrong. The disability must have occurred before age 26

234
Q
  1. A nonqualified deferred compensation plan can be used by an employer as a form of “golden handcuffs” because of which of the following?

A. The forfeiture of the employee’s benefits according to almost any vesting schedule
B. The lack of limitations on the amount of stock grants (such as ISOs)
C. The lack of ERISA requirements
D. The unlimited benefits that can be paid to any one employee

A

A. The forfeiture of the employee’s benefits according to almost any vesting schedule

The plan can have a vesting schedule or a provision for any contingency such as going to work for a competitor. This would bind the employee to the company (handcuff). Answer C is a good answer for the same reason. Answer A is more specific than Answer C as to why it is a “golden handcuff.” Answers B and D would be other reasons for the employee to stay.

235
Q
  1. What is an advantage of buying preferred stock?

A. Warrants can be attached, adding long-term value to the preferred stock.
B. Rights can be attached, adding short-term value to the preferred stock.
C. 50% of the dividends received by a domestic corporation are tax exempt.
D. Both A and C

A

D. Both A and C

Feedback:
Both statements A and C are correct. A subscription warrant may be issued with a bond or preferred stock offering. A warrant is usually issued as a sweetener to make the issue more attractive. It entitles the holder to buy a proportionate amount of the issuer’s common stock at a specific price. Presuming the corporation buying the preferred shares has no ownership in the distributing corporation, seventy percent of the dividends received are tax exempt. Preemptive rights are available to common, rather than to preferred shareholders.
If the corporation wishes to issue more shares, common stockholders have the right to buy these shares before they are offered to the public.

236
Q
  1. Mr. and Mrs. Pell established a Charitable Lead Trust (CLUT) funding it with $2,000,000. The trust assets increased by 7% this year. What percent of the annually revalued principal must the trust pay out to the charitable beneficiary to avoid tax penalties?

A. Not less than 5% of the initial trust value
B. Not less than 5% of the trust assets each year
C. Not less than 7% of the trust assets each year
D. Can be any percentage including skipping the distribution entirely

A

D. Can be any percentage including skipping the distribution entirely

Feedback:
Unlike the CRT (which requires a 5% minimum annual income distribution to the grantor), there are no minimum or maximum required distribution percentages with a charitable lead trust. It is generally permissible to select any percentage, including less than 5%.

237
Q
  1. John and Pamela Underton wrote a check for $500,000 to their daughter, Bunnie, on January 1st of the current year. It was a gift. They wrote the check out of their joint money market account. John called his CPA to inform him that the gift had been made and requested that the CPA prepare two Form 709’s to report the gift splitting. Before the 709 was filed, John died.

Bunnie had not cashed the check as of the date of her father’s death. Given these events, what tax results would be expected?

A. The amount of the check would be considered a gift of a future interest.
B. Because Bunnie had not cashed the check as of the date of her father’s death the transfer will be considered an incomplete gift.
C. The gift will be presumed to have been made by Pamela alone.
D. The transfer will still be presumed as a split gift.

A

D. The transfer will still be presumed as a split gift.

Feedback:
Gifts made before one spouse dies may be split even if that spouse dies before signing the appropriate consent and election on Form 709. On behalf of the deceased spouse, his executor can make the appropriate election. This is not per se an incomplete gift. The check was written from a joint checking account and Pamela (the mother) is still living.

238
Q
  1. Charles divorced Ruth. Under the QDRO that was part of their property settlement, Ruth received one-half of his profit sharing 401(k) plan account having a date of divorce value of $300,000. Those funds are now in Ruth’s own IRA. Before the divorce, Charles and Ruth had a net worth of about $1,500,000. Ruth has about $750,000 which includes a house worth $250,000.
    Ruth has been taking distributions from the IRA which, due to the ongoing withdrawals is declining in value. Concerned, Charles designated Ruth as the beneficiary of his portion of the profit sharing 401(k) plan. Charles recently remarried. If Charles died yesterday, what would you advise Ruth to do if the contingent beneficiaries are their children.

A. Ruth should roll his plan into an inherited IRA.
B. Ruth should roll his plan into a Roth IRA in her name.
C. Ruth should leave the money in the employer’s plan until she needs it.
D. Ruth needs to understand that the only valid beneficiary for Charles’s 401(k) account is his current spouse.

A

A. Ruth should roll his plan into an inherited IRA.

Feedback:
During his first marriage, Charles named Ruth as the beneficiary of his 401(k) account. Nothing indicates that he has changed the beneficiary. Ruth appears to need cash regularly. The Roth conversion would create a large tax bill she likely cannot afford at this time. Because the 401(k) is not a pension plan, there is no requirement that Charles’s current wife be its beneficiary.